Radiation Oncology Study Guide [2nd ed.] 9783030536862, 9783030536879

Now in its second edition, this popular text remains a comprehensive study and review aid for the radiation oncology tra

608 42 4MB

English Pages XIV, 667 [668] Year 2021

Report DMCA / Copyright

DOWNLOAD FILE

Polecaj historie

Radiation Oncology Study Guide [2nd ed.]
 9783030536862, 9783030536879

Table of contents :
Front Matter ....Pages i-xiv
Central Nervous System Tumors: Adult and Pediatric (Blair Murphy, Shushan Rana, Celine Bicquart Ord, Charlotte Dai Kubicky)....Pages 1-46
Skin Cancer and Uveal Melanoma (Reid F. Thompson, Christina Binder, Shushan Rana, Celine Bicquart Ord, John M. Holland)....Pages 47-82
Head and Neck Cancer (Shushan Rana, John M. Holland, Kristina H. Young, Celine Bicquart Ord)....Pages 83-161
Thoracic Cancer (Shushan Rana, Celine Bicquart Ord, Charles R. Thomas Jr.)....Pages 163-219
Tumors of the Mediastinum (Shushan Rana, Celine Bicquart Ord, Charles R. Thomas Jr.)....Pages 221-231
Breast Cancer (Carol Marquez, Pehr Hartvigson, Kiri Cook, Shushan Rana, Celine Bicquart Ord, Charlotte Dai Kubicky)....Pages 233-291
Upper Gastrointestinal Cancers (Vivek Verma, Ethan B. Ludmir, Faisal Siddiqui, Celine Bicquart Ord, Charles R. Thomas Jr.)....Pages 293-324
Lower Gastrointestinal Cancers (Ethan B. Ludmir, Vivek Verma, Shushan Rana, Sravana Chennupati, Celine Bicquart Ord, Charles R. Thomas Jr.)....Pages 325-355
Cancers of the Urinary System (Timur Mitin, Chris Deig, Shushan Rana, Celine Bicquart Ord, Eric K. Hansen)....Pages 357-381
Male Genitourinary Cancers (Timur Mitin, Chris Deig, Shushan Rana, Celine Bicquart Ord, Eric K. Hansen)....Pages 383-441
Gynecological Cancers (Jenna M. Kahn, Shushan Rana, Joseph G. Waller, Celine Bicquart Ord, Subhakar Mutyala)....Pages 443-491
Heme Malignancies (Brandon A. Dyer, Patrick J. Gagnon, Celine Bicquart Ord, Carol Marquez)....Pages 493-539
Soft Tissue Sarcomas and Bone Tumors (Christina Binder, Celine Bicquart Ord, Arthur Hung)....Pages 541-569
Pediatric Non-CNS Tumors (Blair Murphy, Shushan Rana, Celine Bicquart Ord, Carol Marquez)....Pages 571-624
Benign and Metastatic Disease (Eric M. Chang, Celine Bicquart Ord, John M. Holland)....Pages 625-647
Back Matter ....Pages 649-667

Citation preview

Radiation Oncology Study Guide Ravi A. Chandra Celine B. Ord Shushan Rana Eric K. Hansen Charles R. Thomas Jr. Editors Second Edition

123

Radiation Oncology Study Guide

Ravi A. Chandra  •  Celine B. Ord Shushan Rana  •  Eric K. Hansen Charles R. Thomas Jr. Editors

Radiation Oncology Study Guide Second Edition

Editors Ravi A. Chandra Department of Radiation Medicine Knight Cancer Institute, Oregon Health & Science University Portland, OR USA Shushan Rana Radiation Oncologist PeaceHealth Southwest Medical Center Vancouver, WA USA

Celine B. Ord Carson Tahoe Radiation Oncology Associates Carson City, NV USA Eric K. Hansen Providence St Vincent Medical Center The Oregon Clinic Radiation Oncology Portland, OR USA

Charles R. Thomas Jr. Department of Radiation Medicine Knight Cancer Institute, Oregon Health & Science University Portland, OR USA

ISBN 978-3-030-53686-2    ISBN 978-3-030-53687-9 (eBook) https://doi.org/10.1007/978-3-030-53687-9 © Springer Nature Switzerland AG 2021 This work is subject to copyright. All rights are reserved by the Publisher, whether the whole or part of the material is concerned, specifically the rights of translation, reprinting, reuse of illustrations, recitation, broadcasting, reproduction on microfilms or in any other physical way, and transmission or information storage and retrieval, electronic adaptation, computer software, or by similar or dissimilar methodology now known or hereafter developed. The use of general descriptive names, registered names, trademarks, service marks, etc. in this publication does not imply, even in the absence of a specific statement, that such names are exempt from the relevant protective laws and regulations and therefore free for general use. The publisher, the authors and the editors are safe to assume that the advice and information in this book are believed to be true and accurate at the date of publication. Neither the publisher nor the authors or the editors give a warranty, expressed or implied, with respect to the material contained herein or for any errors or omissions that may have been made. The publisher remains neutral with regard to jurisdictional claims in published maps and institutional affiliations. This Springer imprint is published by the registered company Springer Nature Switzerland AG The registered company address is: Gewerbestrasse 11, 6330 Cham, Switzerland

RAC: To my wife Jennie, our children Maya and Nathan, my parents, and my brother and his family for their love, inspiration, and support throughout my education and career to date. To our patients: it is my hope and prayer that the true benefit of this resource will be yours. CBO: To my daughter and muse, Estelle— you inspire me to push forward towards monumental goals while appreciating the small everyday moments. To my husband and best friend, Justin—your endless support and love sustain me. To my mom and personal hero, Farida—you made everything possible for me; I finally understand the depth and power of a mother’s love. SR: To my wife, best friend, and better half, Monica Kishore, who has helped me love others and each other better each day than the last. To my parents, Rajesh and Chhaya Rana, who helped me become a better physician and person by prioritizing excellence in education built on a foundation of care, respect, and love. To my siblings, Ruchit and Shivani Rana, whom I am

honored to call my own through our growth in challenges and joys of this world. EKH: Thanks to all of you who remind me that asking good questions is as valuable as answers! CRT: To my supportive wife, Muriel, our two wonderful children, Julian Franklin and Aurielle Marie, and my parents and siblings for their love and support of my career path. In memory of my mother, Ruth Marie Wilson Thomas, who fought gallantly in the war against cancer and whose prayers have blessed me over the past five decades.

Foreword

The rapid pace of advances in the field of radiation oncology necessitates updated, concise sources of information for both learners and practicing clinicians. Exciting new clinical trials, technological innovations, and creative ways of integrating radiation therapy with other cancer treatments fill this new volume led by Drs. Chandra, Ord, Rana, Hansen, and Thomas. Using a handy question and answer format, expert authors provide comprehensive coverage of both common and rare diseases treated with radiation therapy, literally from head to toe. Not only a useful source of information for those preparing for examinations, this book provides excellent brief overviews of information relevant to key questions that are important to practitioners of radiation oncology and all those physicians who care for patients for whom radiation treatment may have a role. Recent studies have demonstrated that patients often harbor considerable fears and misinformation about radiation therapy. Underuse of this critical component of modern multidisciplinary cancer care appears to relate to its position as a downstream, referral-based specialty and one about which even physicians trained in other oncologic specialties know relatively little. Because radiation oncology has not been given much time in traditional medical school curricula, accessible sources of information about indications, benefits, toxicities, and outcomes are essential. The organization of the book allows readers from other specialties to test themselves and focus on disease sites within their own areas of specialization or interest. It also allows radiation oncologists convenient access to the high-yield current information they require to excel in examinations and meet their patients’ needs. In sum, the leaders of this edition bring diverse perspectives and expertise to draw out unique insights and information of tremendous utility to a wide audience. It is a great honor to have the opportunity to introduce this much needed practical contribution to the literature. Ann Arbor, MI, USA

Reshma Jagsi, MD, DPhil

vii

Preface

Radiation oncology continues to evolve at breakneck speed, with our clinical practice becoming ever more multidisciplinary and evidence-based with each passing year. Now in its second edition, this comprehensive book has targeted the needs of medical students and residents/fellows studying for exams, as well as independent practitioners aiming to improve their knowledge as they prepare for recertification requirements, such as the American Board of Radiology’s new Online Longitudinal Assessment (OLA) and written recertification exam. It is our hope that this book will continued to be a valuable resource to many within our field. The questions and answer topics in this edition have been revised extensively, with particular attention to recent developments in each subsite. Answer explanations have been overhauled to be more concise, bulleted, and high-yield to aid in quick study and review. We hope that our questions lead to more questions from our readers, which prompts further learning. While covering the breadth of radiation oncology is difficult, our expectation is that this review addresses the majority of the important concepts of radiation treatment for each disease site. Beyond the concisely summarized studies in each of the rationales, readers are encouraged to review the references listed at the end of each answer’s rationale (primary literature and aforementioned sources) for further information. As trials are updated and new studies published, our long-term goal is to regularly update this high-quality resource. We sincerely thank the contributors for their dedication, hours of hard work, and excellent chapters. We would also like to acknowledge Margaret Moore and the team at Springer for their time and expertise that have enabled this project to come to fruition. Portland, OR, USA Carson City, NV, USA Vancouver, WA, USA Carson City, NV, USA Portland, OR, USA

Ravi A. Chandra, MD, PhD Celine B. Ord, MD Shushan Rana, MD Eric K. Hansen, MD Charles R. Thomas Jr., MD

ix

Contents

1 Central Nervous System Tumors: Adult and Pediatric������������������������    1 Blair Murphy, Shushan Rana, Celine Bicquart Ord, and Charlotte Dai Kubicky 2 Skin Cancer and Uveal Melanoma��������������������������������������������������������   47 Reid F. Thompson, Christina Binder, Shushan Rana, Celine Bicquart Ord, and John M. Holland 3 Head and Neck Cancer����������������������������������������������������������������������������   83 Shushan Rana, John M. Holland, Kristina H. Young, and Celine Bicquart Ord 4 Thoracic Cancer��������������������������������������������������������������������������������������  163 Shushan Rana, Celine Bicquart Ord, and Charles R. Thomas Jr. 5 Tumors of the Mediastinum��������������������������������������������������������������������  221 Shushan Rana, Celine Bicquart Ord, and Charles R. Thomas Jr. 6 Breast Cancer ������������������������������������������������������������������������������������������  233 Carol Marquez, Pehr Hartvigson, Kiri Cook, Shushan Rana, Celine Bicquart Ord, and Charlotte Dai Kubicky 7 Upper Gastrointestinal Cancers ������������������������������������������������������������  293 Vivek Verma, Ethan B. Ludmir, Faisal Siddiqui, Celine Bicquart Ord, and Charles R. Thomas Jr. 8 Lower Gastrointestinal Cancers ������������������������������������������������������������  325 Ethan B. Ludmir, Vivek Verma, Shushan Rana, Sravana Chennupati, Celine Bicquart Ord, and Charles R. Thomas Jr. 9 Cancers of the Urinary System��������������������������������������������������������������  357 Timur Mitin, Chris Deig, Shushan Rana, Celine Bicquart Ord, and Eric K. Hansen

xi

xii

Contents

10 Male Genitourinary Cancers������������������������������������������������������������������  383 Timur Mitin, Chris Deig, Shushan Rana, Celine Bicquart Ord, and Eric K. Hansen 11 Gynecological Cancers����������������������������������������������������������������������������  443 Jenna M. Kahn, Shushan Rana, Joseph G. Waller, Celine Bicquart Ord, and Subhakar Mutyala 12 Heme Malignancies����������������������������������������������������������������������������������  493 Brandon A. Dyer, Patrick J. Gagnon, Celine Bicquart Ord, and Carol Marquez 13 Soft Tissue Sarcomas and Bone Tumors������������������������������������������������  541 Christina Binder, Celine Bicquart Ord, and Arthur Hung 14 Pediatric Non-CNS Tumors��������������������������������������������������������������������  571 Blair Murphy, Shushan Rana, Celine Bicquart Ord, and Carol Marquez 15 Benign and Metastatic Disease ��������������������������������������������������������������  625 Eric M. Chang, Celine Bicquart Ord, and John M. Holland Index������������������������������������������������������������������������������������������������������������������  649

Contributors

Christina Binder, MD  Spears Cancer Center, Grants Pass, OR, USA Department of Radiation Medicine, Oregon Health Science University, Portland, OR, USA Eric Chang, MD  Department of Radiation Oncology, University of California Los Angeles, Los Angeles, CA, USA Department of Radiation Medicine, Oregon Health & Science University, Portland, OR, USA Sravana  Chennupati, Berkeley, CA, USA

MD  Alta

Bates

Comprehensive

Cancer

Center,

Kiri  Cook, MD  Department of Radiation Medicine, Oregon Health Science University, Portland, OR, USA Chris Deig, MD  Oregon Health and Science University, Portland, OR, USA Brandon  A.  Dyer, MD  Radiation Therapy, Gynecologic Oncology & Brachytherapy Services, Department of Radiation Oncology | UW Medicine, Seattle, WA, USA Patrick J. Gagnon, MD  Southcoast Health Cancer Center, Fairhaven, MA, USA Eric K. Hansen, MD  Providence St Vincent Medical Center, The Oregon Clinic Radiation Oncology, Portland, OR, USA Pehr Hartvigson, MD  Department of Radiation Medicine, Oregon Health Science University, Portland, OR, USA John M. Holland, MD  Department of Radiation Medicine, Oregon Health Science University, Portland, OR, USA Arthur  Hung, MD  Department of Radiation Medicine, Oregon Health and Sciences University, Portland, OR, USA Jenna M. Kahn, MD  Oregon Health & Science University, Portland, OR, USA xiii

xiv

Contributors

Charlotte  Dai  Kubicky, MD, PhD  Department of Radiation Medicine, Oregon Health Science University, Portland, OR, USA Department of Radiation Oncology, Sutter Health, Roseville, CA, USA Ethan  B.  Ludmir, MD  Department of Radiation Oncology, The University of Texas MD Anderson Cancer Center, Houston, TX, USA Carol  Marquez, MD  Stanford School of Medicine, Stanford University, San Jose, CA, USA Timur Mitin, MD, PhD  Oregon Health and Science University, Portland, OR, USA Blair  Murphy, MD  Oregon Health & Science University in Portland in the U.S. State of Oregon, Portland, OR, USA Subhakar Mutyala, MD  Radiation Oncology, Phoenix, AZ, USA Celine Bicquart Ord, MD  Carson Tahoe Radiation Oncology Associates, Carson City, NV, USA Shushan  Rana, MD  Radiation Oncologist, PeaceHealth Southwest Medical Center, Vancouver, WA, USA Faisal Siddiqui, MD, PhD  Compass Oncology, Vancouver, WA, USA Charles R. Thomas Jr., MD  Department of Radiation Medicine, Oregon Health Science University, Portland, OR, USA Reid F. Thompson, MD, PhD  Department of Radiation Medicine, Oregon Health Science University, Portland, OR, USA Vivek Verma, MD  Department of Radiation Oncology, The University of Texas MD Anderson Cancer Center, Houston, TX, USA Joseph G. Waller, MD  Radiation Oncology, Saint Charles, MO, USA Kristina  H.  Young, MD, PhD  The Oregon Clinic  – Radiation Oncology, Portland, OR, USA

Chapter 1

Central Nervous System Tumors: Adult and Pediatric Blair Murphy, Shushan Rana, Celine Bicquart Ord, and Charlotte Dai Kubicky

Questions 1. Which of the following subgroups of medulloblastoma has the highest rate of metastatic recurrence? A. WNT B. SHH C. Group 3 D. Group 4. 2. What type of intracranial tumor is most commonly seen with neurofibromatosis type 2? A. Schwannoma B. Meningioma C. Astrocytoma D. Oligodendroglioma

B. Murphy (*) Oregon Health & Science University in Portland in the U.S. State of Oregon, Portland, OR, USA e-mail: [email protected] S. Rana Radiation Oncologist, PeaceHealth Southwest Medical Center, Vancouver, WA, USA C. B. Ord Carson Tahoe Radiation Oncology Associates, Carson City, NV, USA C. D. Kubicky Department of Radiation Medicine, Oregon Health Science University, Portland, OR, USA © Springer Nature Switzerland AG 2021 R. A. Chandra et al. (eds.), Radiation Oncology Study Guide, https://doi.org/10.1007/978-3-030-53687-9_1

1

2

B. Murphy et al.

3. All of the following are correct regarding Turcot’s syndrome except: A. There is an association with familial polyposis of the colon. B. There is an association with medulloblastoma. C. There is an association with high-grade glioma. D. There is an association with low-grade glioma. 4. What is the approximate rate of >G2 symptomatic brainstem necrosis for pediatric CNS tumors treated with proton therapy? A. 0.5% B. 2% C. 5% D. 12% 5. Which gene fusion portends a poor prognosis for pediatric patients with supratentorial ependymoma? A. RELA B. YAP1 C. EWSR1-FLI1 D. PAX3-FOXO1 6. ACNS 0331 investigated radiation field reduction and radiation dose reduction for patients with standard-risk medulloblastoma. What two important findings resulted from this trial? A. Tumor bed boost had equivalent outcomes to posterior fossa boost fields, but reduced-dose craniospinal irradiation for younger children resulted in worse outcomes. B. Tumor bed boost had worse outcomes than posterior fossa boost, and reduced-dose CSI for young patients showed equivalent outcomes. C. Tumor bed boost had equivalent outcomes to posterior fossa boost, and reduced-dose CSI for young patients showed equivalent outcomes. D. Tumor bed boost had improved outcomes over posterior fossa boost, but reduced-dose CSI for young patients showed inferior outcomes. 7. What mutation is now associated with the diagnosis of diffuse midline glioma per the new 2016 World Health Organization (WHO) released their updated classifications? A. MGMT B. RELA C. H3K27M D. IDH1

1  Central Nervous System Tumors: Adult and Pediatric

3

8. A young child with medulloblastoma undergoes MRI spine and is found to have gross nodular seeding in the spinal arachnoid space. According to the Chang’s operative staging system, what is their metastatic stage? A. M1 B. M2 C. M3 D. M4 9. Neurofibromatosis type 1, von Recklinghausen’s disease, is most commonly associated with what type of intracranial tumor? A. Meningioma B. Astrocytoma C. Glioblastoma D. Oligodendroglioma 10. All of the following are tumors associated with tuberous sclerosis except: A. Hemangioblastoma B. Angiofibromas C. Hamartomas D. Subependymal giant cell tumors 11. Cerebellar, brainstem, and spinal cord hemangioblastomas are frequently seen in which disorder? A. Cowden’s syndrome B. Turcot’s syndrome C. Von Hippel-Lindau disease D. Gorlin’s syndrome 12. In which of the following tumors does survival increase with increasing age? A. Glioblastoma multiforme B. Oligodendroglioma C. Anaplastic astrocytoma D. Brainstem glioma 13. MDM2 amplification/overexpression is a genetic event frequently seen in what tumor? A. Secondary glioblastoma multiforme. B. Primary glioblastoma multiforme C. Anaplastic astrocytoma D. Oligodendroglioma

4

B. Murphy et al.

14. Compared to IDH-wild-type glioblastoma, IDH-mutant glioblastoma: A. Comprise the majority of glioblastoma cases B. Have worse overall survival and progression-free survival C. Have a temporal lobe distribution D. Most likely arise from a lower-grade glioma precursor lesion 15. Deletion of 1p and 19q in oligodendroglioma is associated with: A. Better response to therapy but similar progression-free and overall survival B. Better response to therapy with improved progression-free but similar overall survival C. Similar response to therapy with improved progression-free and overall survival D. Better response to therapy with improved progression-free and overall survival 16. Methylation of the O6-methylguanine-DNA methyltransferase, MGMT, promoter is associated with: A. Decreased MGMT expression and improved overall survival. B. Decreased MGMT expression and decreased overall survival C. Increased MGMT expression and improved overall survival D. Increased MGMT expression with no effects on overall survival 17. The most common presenting symptom of low-grade astrocytoma is: A. Headache B. Weakness C. Personality change D. Seizure 18. Which one of the following is NOT a negative risk factor in the prognostic scoring system of low-grade gliomas developed by Pignatti and colleagues? A. Astrocytoma histology subtype B. Tumor crossing the midline C. Age 40 years or less D. Presence of neurological deficit before surgery 19. All of the following are true regarding the EORTC 22844 “Believer’s Trial” in low-grade glioma except: A. Only 25% of patients underwent gross total resection. B. 45 Gy was the radiation dose of one arm in the study. C. 50.4 Gy was the radiation dose of one arm in the study. D. Overall survival did not improve with dose escalation.

1  Central Nervous System Tumors: Adult and Pediatric

5

20. According to the long-term results of EORTC 22845 in which early versus delayed radiotherapy for resected low-grade gliomas was compared, which of the following is NOT true? A. Progression-free survival improved in the early radiotherapy group. B. Overall survival was not different between the early versus delayed groups. C. Seizures were better controlled at 1 year in the delayed radiotherapy group. D. Total dose of radiotherapy given was 54 Gy. 21. In the EORTC 26951 trial, patients with low-grade glioma were randomized after surgery and radiation to +/− chemotherapy. Which statement regarding this trial is false? A. Chemotherapy consisted of PCV (procarbazine, lomustine, and vincristine). B. The initial published results showed only a progression-free survival benefit. C. Follow-up long-term results still showed no overall survival benefit. D. Median survival has not yet been reached in those with 1p19q deletion that received adjuvant PCV chemotherapy. 22. Regarding magnetic resonance spectroscopy (MRS), which of the following is NOT true? A. Brain tumors are characterized by an elevated choline to creatine ratio. B. Radiation change on MRS is characterized by decreased choline levels. C. The MR spectrum of brain tumors is characterized by an increasing Hunter’s angle from left to right. D. Radiation change on MRS is characterized by decreased N-acetylaspartate. 23. In a four-arm ECOG/RTOG trial for high-grade gliomas comparing a standard 60 Gy to 3 experimental arms, which is NOT true? A. A 10 Gy boost in addition to the standard 60 Gy WBRT improved survival. B. Forty- to sixty-year-old patients in the 60 Gy + BCNU arm had significantly increased survival. C. The 60  Gy  +  methyl-CCNU and DTIC arm was more toxic than the 60 Gy + BCNU arm. D. Age was found to be the most important prognostic factor. 24. In a dose escalation study of malignant gliomas performed by Lee et al. at the University of Michigan, which of the following is NOT true? A. Dose-limiting toxicity was not seen. B. There was no trend in improved overall survival despite dose escalation. C. Despite dose escalation, the majority of tumors progressed “in-field.” D. In addition to local progression, there was extensive distant progression.

6

B. Murphy et al.

25. Regarding RTOG 90–05, a radiosurgery dose-finding study, which of the following is true? A. Dose-limiting toxicity was defined as irreversible grade 2 neurotoxicity. B. The maximum tolerated dose for tumors 2–3 cm was 15 Gy. C. Patients with gliomas are more likely to fail locally compared to those with metastases. D. Approximately two-thirds of patients failed locally. 26. Regarding brachytherapy in the brain for malignant glioma, all of the following are true except: A. In a Brain Tumor Cooperative Group study, there was no survival benefit seen with the addition of a brachytherapy boost. B. The GliaSite RT system uses an intracavitary device filled with an 192Ir solution. C. In using an intracavitary balloon for brachytherapy, there is not a true point source. D. Smaller balloon fill volumes may result in higher doses at the balloon surface, possibly leading to increased radiation necrosis. 27. All of the following regarding temozolomide are true except: A. It is an alkylating agent with a half-life of minutes. B. Hypermethylation of the MGMT promoter leads to increased sensitivity to temozolomide. C. It was first approved as a single-agent in recurrent glioma. D. Concurrent with radiation for newly diagnosed glioblastoma, it is given daily at 150 mg/m2. 28. What were the 5-year overall survivals with and without temozolomide, respectively, reported in the latest (2009) update of the Stupp trial? A. 10%, 2% B. 2%, 10% C. 27%, 10% D. 10%, 27% 29. According to the 2009 update of the Stupp trial, radiation +/− temozolomide for glioblastoma, what was the strongest predictor of outcome? A. Extent of resection B. Age C. Performance status D. MGMT status 30. Which of the following targets/therapies for glioblastoma multiforme is incorrect? A. EGFR/erlotinib B. Farnesyl transferase/tipifarnib C. EGFR/trastuzumab D. VEGF/bevacizumab

1  Central Nervous System Tumors: Adult and Pediatric

7

31. What was the median survival in those elderly patients treated with radiotherapy as opposed to best supportive care, as reported by Bauman et al.? A. 4 months B. 6 months C. 8 months D. 10 months 32. Comparing standard fractionation to hypofractionated radiotherapy for poor prognosis GBM patients aged 60 years or older, Roa et al. found: A. Improved survival in the standard fractionation arm. B. Improved survival in the hypofractionated arm. C. Those receiving standard fractionation were more likely to require an increase in steroids posttreatment. D. More patients in the hypofractionated arm did not complete their planned treatment course. 33. What was the prescribed dose of radiotherapy in the randomized glioblastoma multiforme in the elderly study, as reported by Keime-Guibert et al.? A. 30 Gy B. 45 Gy C. 50.4 Gy D. 60 Gy 34. The addition of temozolomide to hypofractionated radiation for GBM in the elderly has shown following: A. No change in overall survival in patients with methylated MGMT promoter B. Improved progression-free survival with no change in overall survival C. Improved progression-free survival and overall survival with improved quality of life D. Improved progression-free survival and overall survival with no difference in quality of life 35. All of the following tumors have a propensity for distant dissemination except: A. CNS germ cell tumors B. Ependymoma C. Choroid plexus tumors D. Meningioma 36. Maximal resection is an important initial component in therapy in all of the following except: A. Glioblastoma multiforme. B. Meningioma C. CNS germinoma D. Chondrosarcoma

8

B. Murphy et al.

37. All of the following are risk factors for the development of meningioma except: A. Female sex. B. Neurofibromatosis type 2 C. Prior exposure to ionizing radiation D. Asian race 38. What pathological feature is frequently present in meningioma? A. Psammoma bodies B. Rosenthal fibers C. Pseudorosettes D. Gemistocytes 39. Which of the following pathways of spread of meningioma is least common? A. Growth along the base of skull B. Growth along meningeal surfaces C. Growth with invasion into the brain D. Growth with invasion of adjacent dural or superior sagittal sinuses 40. Which statement is incorrect regarding 5-year progression-free survival (5-year PFS) of meningioma, as reported by Goldsmith et al.? A. Gross totally resected benign meningioma has a 5-year PFS of >80%. B. Subtotally resected benign meningioma has a 5-year PFS of approximately 50%. C. The addition of radiation after subtotally resected benign meningioma raises progression-free survival to >85%. D. The addition of radiation after resection of malignant meningioma raises progression-free survival to >70%. 41. Which is the most typical appearance of meningioma on magnetic resonance imaging? A. Isointense on pre-contrast T1, no enhancement with gadolinium contrast B. Isointense on pre-contrast T1, uniform enhancement with gadolinium contrast C. Hyperintense on pre-contrast T1, no enhancement with gadolinium contrast D. Hyperintense on pre-contrast T1, uniform enhancement with gadolinium contrast 42. In a low-grade oligodendroglioma, which of the following radiographic patterns would most typically be noted on magnetic resonance imaging? A. Enhances on T1 after gadolinium administration, hypointense on T1 pre-contrast B. Non-enhancing on T1 after gadolinium administration, hypointense on T1 pre-contrast C. Non-enhancing on T1 after gadolinium administration, hyperintense on T1 pre-contrast D. Enhances on T1 after gadolinium administration, hyperintense on T1 pre-contrast

1  Central Nervous System Tumors: Adult and Pediatric

9

43. All of the following are rationales for the use of stereotactic radiosurgery to treat meningioma except: A. Lesions are typically well circumscribed. B. Alpha/beta ratio is similar to that of late-reacting tissue. C. Improvement in local control of subtotally resected lesions. D. Treatment side effects are independent of tumor location. 44. In treating parasellar meningiomas with SRS, what dose limit to the optic apparatus is suggested to reduce the risk of optic neuropathy? A. 12 Gy B. 8 Gy C. 6 Gy D. 5 Gy 45. Rosenthal fibers are a pathognomonic feature of which tumor type? A. Pleomorphic xanthoastrocytomas B. Oligodendrogliomas C. Germinomas D. Pilocytic astrocytomas 46. Which one of the following tumor/pathologic feature pairings is incorrect? A. Glioblastoma/necrosis B. Medulloblastoma/pseudorosettes C. Craniopharyngioma/cystic areas D. Oligodendroglioma/fried egg appearance 47. Comparing the incidence in adults and children, ependymomas tend to be: A. Intracranial in adults and supratentorial in children. B. Spinal in adults and infratentorial in children C. Spinal in adults and supratentorial in children D. Intracranial in adults and infratentorial in children 48. Which of the following subtypes of ependymoma has the worst prognosis? A. Myxopapillary. B. Anaplastic C. Solid squamous papillary D. Cellular 49. Which of the following statements about ependymoma is false? A. Gross total resection is achieved more commonly in infratentorial compared to supratentorial tumors. B. The predominant pattern of failure is local. C. The extent of resection is the most important prognostic factor. D. Radiotherapy dose is typically 59.4 Gy.

10

B. Murphy et al.

50. Which of the following is not a common site of presentation of chordoma? A. Base of skull B. Lower cervical spine C. Lumbar spine D. Sacrum 51. Protons are used in place of photons for some chordomas due to all but which of the following: A. Steep dose gradients of protons. B. Excellent 5-year local control rates C. Higher RBE of protons D. Proximity of tumor to critical structures 52. All of the following are prognostic factors of the RTOG RPA classification for high-grade glioma developed by Curran et al. except: A. Age > 60 B. Histology C. Performance status D. Extent of resection 53. All are true regarding the RTOG RPA analysis for glioblastoma multiforme developed by Li et al. except: A. It includes only four variables, compared to the prior RPA analysis, which included six. B. Classes V and VI are combined due to indistinguishable survival. C. The extent of resection is no longer one of the four variables in the new RPA analysis. D. This new RPA analysis will be used for future RTOG glioblastoma multiforme trials. 54. All of the following are true regarding the RTOG study 94–02 (Cairncross et al.) looking at outcomes in pure and mixed oligodendroglioma receiving postoperative radiotherapy +/− PCV chemotherapy except: A. Progression-free survival was improved with the addition of PCV chemotherapy. B. Overall survival was not improved with the addition of PCV chemotherapy. C. Those tumors with 1p and 19q allelic loss had median survival times >7 years. D. There was no increased toxicity with the addition of PCV chemotherapy to radiotherapy. 55. How many mitoses per 10 high-powered fields constitute anaplastic meningioma? A. 5 B. 10 C. 15 D. 20

1  Central Nervous System Tumors: Adult and Pediatric

11

56. What is the grading system used to describe completeness of resection of meningiomas? A. Simpson B. Brooker C. Bloom-Richardson D. Kadish 57. What is the most common site for a CNS germ cell tumor? A. Anterior pituitary B. Posterior pituitary C. Pineal gland D. Suprasellar region 58. What syndrome is commonly seen with pineoblastoma? A. Eaton-Lambert B. Parinaud’s C. Limbic encephalitis D. Cushing’s 59. Which of the following pineal tumors has the LEAST propensity for craniospinal dissemination? A. Pineoblastoma B. Germinoma C. Non-germinomatous germ cell tumor D. Pineocytoma 60. All of the following statements regarding hemangioblastomas and hemangiopericytomas are true except: A. While hemangioblastomas are indolent and slow growing, hemangiopericytomas are locally aggressive. B. Hemangioblastomas and hemangiopericytomas are both associated with Von Hippel-Lindau. C. Hemangiopericytoma, in contrast to hemangioblastoma, shows a propensity for local recurrence after resection. D. Surgical resection is the primary therapy for both tumors. 61. Regarding craniopharyngiomas, all of the following are true except: A. They are derived from remnants of Rathke’s pouch. B. There is a bimodal incidence. C. Calcification is infrequently seen. D. Diabetes insipidus is a frequent complication of surgery. 62. What percent of pituitary adenomas are nonsecretory? A. 30% B. 40% C. 50% D. 60%

12

B. Murphy et al.

63. Which of the following hormones is secreted by the neurohypophysis? A. ADH B. ACTH C. GH D. TSH 64. Which one of the following hormones secreted by a pituitary adenoma has no effective medical therapy? A. Prolactin B. Thyroid-stimulating hormone C. Adrenocorticotropic hormone D. Growth hormone 65. After radiation to the pituitary axis, which is most likely to be the first hormone to be affected? A. Gonadotropin-releasing hormone B. Thyroid-stimulating hormone C. Adrenocorticotropic hormone D. Growth hormone 66. What is the most caudal structure transmitted through the cavernous sinus? A. CN III B. CN V1 (V one) C. CN V2 D. CN VI (six) 67. What is the most frequent primary ocular tumor? A. Melanoma B. Lymphoma C. Retinoblastoma D. Optic nerve sheath meningioma 68. Which of the following primary ocular tumors occurs more frequently in women than men? A. Melanoma B. Lymphoma C. Retinoblastoma D. Optic nerve sheath meningioma 69. Which is the most common tumor type to metastasize to the orbit? A. Non-small-cell lung cancer B. Hepatocellular carcinoma C. Breast cancer D. Small-cell lung cancer

1  Central Nervous System Tumors: Adult and Pediatric

13

70. Which of the following collaborative ocular melanoma study (COMS) classifications is incorrect? A. Small melanoma: 1–3 mm thick B. Small melanoma: 16 mm in largest tumor dimension 71. Which of the following is true regarding the COMS study for large choroidal melanomas? A. Patients were randomized to enucleation versus plaque brachytherapy. B. Patients were randomized to enucleation versus external beam radiotherapy. C. Patients were randomized to enucleation +/− plaque brachytherapy. D. Patients were randomized to enucleation +/− preoperative external beam radiotherapy. 72. Acoustic neuromas most commonly affect which cranial nerve? A. Cranial nerve V B. Cranial nerve VII C. Cranial nerve VIII D. Cranial nerve IX 73. Which of the following is not an embryonal CNS tumor? A. Medulloblastoma B. Atypical teratoid/rhabdoid tumor C. Germinoma D. Pineoblastoma 74. What is the typical radiotherapy regimen for embryonal CNS tumors? A. None B. Craniospinal irradiation C. Focal radiation D. Whole ventricular radiation 75. Which of the following molecular subtypes of medulloblastoma has the best prognosis? A. Wnt-pathway-associated subtype B. Hh-pathway-associated subtype C. c-MYC-pathway-associated subtype D. Other subtypes 76. What is the superior border of the posterior fossa? A. Clivus B. Tentorium cerebelli C. Calvarium D. Occipital bone

14

B. Murphy et al.

77. What would be the recommended radiotherapy regimen for standard-risk medulloblastoma? A. Craniospinal radiation to 36 Gy, followed by a boost to the posterior fossa to a total of 54 Gy in combination with multiagent chemotherapy B. Craniospinal radiation to 54  Gy in combination with multiagent chemotherapy C. Craniospinal radiation to 23.4  Gy, followed by a boost to the posterior fossa to a total of 54 Gy in combination with multiagent chemotherapy D. Craniospinal radiation to 23.4  Gy in combination with multiagent chemotherapy 78. In a St. Jude Children’s Research trial prospectively evaluating focal treatment of the tumor bed in medulloblastoma patients, as opposed to treatment of the entire posterior fossa, all of the following are true except: A. Dose to the entire posterior fossa was 36  Gy, followed by a conformal tumor bed boost to 54 Gy. B. 3-year posterior fossa failure was 6.3%. C. Chemotherapy was given after radiation. D. Dose reduction to the posterior fossa was over 10%. 79. Which is not a component of therapy for children G3 toxicity: 1.3%. –– Fatal brainstem toxicity was 0.4%. Indelicato et al. • Smaller cohort of 60 patients treated with photon therapy • Results –– Symptomatic brainstem toxicity: 6.7% –– >G3 toxicity: 3.3% –– Fatal toxicity of 1.7% • Average dose of patients who experienced symptomatic brainstem necrosis, 55.7 Gy. Indelicato DJ, et al. Incidence and dosimetric parameters of pediatric brainstem toxicity following proton therapy. Acta Oncol. 2014;53:1298–304. Haas-Kogan DE, et al. National Cancer Institute Workshop on proton therapy for children: considerations regarding brainstem injury. IJORBP. 2018;101(1):152–68. 5. The correct answer is A. Gene fusions • RELA fusion gene –– Results in activation of NFKB pathologic pathway –– Detected by FISH –– Tend to have tumors in the supratentorial location –– Compared to its YAP1 fusion counterpart ◦◦ RELA fusion + ependymomas have worse 5-year OS (75% vs 100%) and PFS (29% vs 66%) • EWRS1-FlI1 fusion –– Associated with poor prognosis in Ewing sarcoma • PAX3-FOXO1 fusion –– Associated with poor prognosis in alveolar rhabdomyosarcoma Pajtier KW, et  al. Molecular classification of ependymal tumors across all CNS compartments, histopathological grades, and age groups. Cancer Cell. 2015;27(5):728–43. Dupain C, et  al. Relevance of fusion genes in pediatric cancers: toward precision medicine. Mol Ther Nucleic Acids. 2017;6:315–26. Gajjar AM, et al. Pediatric brain tumors: innovative genomic information is transforming the diagnostic and clinical landscape. JCO. 2015;33(27):2986–98.

1  Central Nervous System Tumors: Adult and Pediatric

21

6. The correct answer is A. ACNS 0331 • Standard-risk medulloblastoma patients • Randomization –– First randomization ◦◦ Involved-field radiotherapy (IFRT) of tumor bed ◦◦ Standard posterior fossa radiotherapy (PFRT) –– Second randomization of children ages 3–7 ◦◦ Standard dose CSI with 23.4 Gy ◦◦ Reduced-dose 18 Gy • Results –– IFRT non-inferior to PFRT ◦◦ 5-yr EFS 82.2% IFRT vs 80.8% PFRT ◦◦ 5-yr OS 84.1% IFRT vs 85.2% PFRT –– Reduced-dose CSI inferior to standard dose CSI ◦◦ 5-yr EFS 82.6% 23.4 Gy vs 72.1% 18 Gy ◦◦ 5-yr OS 85.9% 23.4 Gy vs 78.1%. Michalski JM, et al. Results of COG ACNS0331: a phase III Trial of InvolvedField Radiotherapy (IFRT) and Low Dose Craniospinal Irradiation (LD-CSI) with chemotherapy in average-risk medulloblastoma: a report from the Children’s Oncology Group. IJORBP. 2016;96:937–8. 7. The correct answer is C. HK327M mutation • Associated with a Lys 27-to-methionine (K27M) mutation at one allele of H3F3A. • H3.3 is associated with a worse prognosis than H3.1. • Carries dismal prognosis regardless of tumor location (mean overall survival of 0.73 years). Khuong-Quang, et al. K27M mutation in histone H3.3 defines clinically and biologically distinct subgroups of pediatric diffuse intrinsic pontine gliomas. Acta Neuropathol. 2012;124(3):439–47. Chan KM, et  al. The histone H3.3K27M mutation in pediatric glioma reprograms H3K27 methylation and gene expression. Genes Dev. 2013;27(9):985–90. 8. The correct answer is C. Chang’s operative staging system: • M0—no metastasis • M1—tumor cells present CSF • M2—nodular seeding in the cerebellar or cerebral subarachnoid space or in the third or lateral ventricle • M3—metastasis in the spinal subarachnoid space • M4—metastases outside the cerebrospinal axis Dufour C, et al. Metastatic medulloblastoma in childhood: Chang’s classification revisited. Int J Surg Oncol. 2012;2012:245385.

22

B. Murphy et al.

9. The correct answer is B. Intracranial astrocytomas of any grade can be seen. Pilocytic astrocytomas (WHO I) are particularly common. Brown PD, Shaw EG, van den Bendt MJ. Low-grade gliomas. In: Gunderson LL, Tepper JE, editors, Clinical radiation oncology. 3rd ed. Philadelphia: Elsevier Saunders; 2012. p. 443–59. 10. The correct answer is A. Tuberous sclerosis associated with • Angiofibromas • Hamartomas • Subependymal giant cell tumors Von Hippel-Lindau associated with hemangioblastomas Northrup H, Koenig MK, Pearson DA, et al. Tuberous sclerosis complex. 1999 Jul 13 [Updated 2018 Jul 12]. In: Adam MP, Ardinger HH, Pagon RA, et al., editors. GeneReviews® [Internet]. Seattle (WA): University of Washington, Seattle. p. 1993–2019. 11. The correct answer is C. Genetic syndromes and CNS malignancies • Von Hippel-Lindau –– Autosomal dominant disorder –– Caused by tumor suppressor gene on chromosome 3 –– Cerebellar, brainstem, and spinal cord hemangioblastomas • Cowden’s syndrome (multiple hamartoma syndrome) –– Autosomal dominant inherited syndrome –– Mutation in the PTEN gene on arm 10q –– Hamartomatous neoplasms of the skin, mucosa, GI tract, bones, CNS, eyes, and GU tract ◦◦ Skin involved in 90–100% of cases ◦◦ Thyroid involved in 66% of cases • Turcot’s syndrome –– Associated with the biallelic DNA mismatch repair mutations –– Association between familial polyposis of colon and brain tumors (medulloblastoma, malignant glioma) • Gorlin’s syndrome –– Autosomal dominant inherited disorder –– Also known as nevoid basal cell carcinoma syndrome –– Caused by mutations in the PTCH gene on chromosome 9q –– Associated with development of basal cell carcinomas –– Benign odontogenic keratocysts of the jaw seen in 75% of patients, usually occurring at a young age –– Less commonly seen: ovarian or cardiac fibromas and/or medulloblastomas (seen in children ≤2 years) Liaw D, et al. Germline mutations of the PTEN gene in Cowden disease, an inherited breast and thyroid cancer syndrome. Nat Genet. 1997;16(1):64–7. Lloyd KM II, Dennis M. Cowden’s disease. A possible new symptoms complex

1  Central Nervous System Tumors: Adult and Pediatric

23

with multiple system involvement. Ann Intern Med. 1963;58:136–42. Turcot J, et al. Malignant tumors of the central nervous system associated with familial polyposis of the colon and bright green urine which may be related to vegetables in the diet: report of two cases. Dis Colon Rectum. 1959; 2:465–8. Gorlin R, Goltz R. Multiple nevoid basal cell epithelioma, jaw cysts and bifid rib. A syndrome. N Engl J Med. 1960;262(18):908–12. 12. The correct answer is D.  Whereas survival decreases with increasing age in glioblastoma multiforme, oligodendroglioma, and anaplastic astrocytoma, it improves in brainstem glioma with increasing age. Freeman CR, Farmer JP. Pediatric brain stem gliomas: a review. IJROBP. 1998; 40:265–71. Guillamo JS, Monjour A, Taillandier L, et al. Brainstem gliomas in adults: prognostic factors and classification. Brain 2001; 124:2528–39. 13. The correct answer is B. MDM2 overexpression is frequently seen in primary glioblastoma, while loss of 19q and IDH1 positivity is associated with secondary glioblastoma. Genetics of glioma progression and the definition of primary and secondary glioblastoma. Brain Pathol. 1997; 7:1131–36. Von Diemling A, Korshunov A, Hartmann C. The next generation of glioma biomarkers: MGMT methylation, BRAF mutations, and IDH1 mutations. Brain Pathol. 2011; 1:74–87. 8. The correct answer is D. 1p and 19q deletion is an established prognostic factor for anaplastic oligodendrogliomas. Hoang-Xuan et al. described their prospective trial results, which showed loss of 1p correlated with objective tumor response. In analyzing 1p19 status of 100 low-grade glioma patients enrolled on two North Central Cancer Treatment Group Protocols, Buckner et al. found that 1p and 19q deletions were associated with superior 5-year progression-free and overall survival. Hoang-Xuan K, Capelle L, et al. Temozolomide as initial treatment for adults with low-grade oligodendrogliomas or oligoastrocytomas and correlation with chromosome 1p deletions. J Clin Oncol. 2004; 22:3133–8. Bucker JC, Ballman KV, et al. Diagnostic and prognostic significance of 1p and 19q deletions in patients (pts) with low-grade oligodendroglioma and astrocytoma. J Clin Oncol. 2005; 23:1145. 14. The correct answer is D. 2016 World Health Organization Classification of Tumors of the Central Nervous System • GBM divided into: –– IDH-wild-type ◦◦ 90% of cases ◦◦ Typically diagnosed as a primary or de novo glioblastoma ◦◦ Predominates in patients over 55 years of age • IDH-mutant –– 10% of cases –– Correspond to “secondary glioblastoma” –– History of prior lower-grade diffuse glioma

24

B. Murphy et al.

–– Preferentially arises in younger patients –– location is typically in the frontal lobes • NOS subtype is reserved for tumors with inconclusive IDH status. Louis DN, et al. The 2016 World Health Organization classification of tumors of the central nervous system: a summary. Acta Neuropathol. 2016;131(6):803–20 15. The correct answer is D. • 1p and 19q deletion is an established prognostic factor for anaplastic oligodendrogliomas. –– Hoang-Xuan et al. described their prospective trial results: ◦◦ Loss of 1p correlated with objective tumor response –– Buckner et al. ◦◦ 1p and 19q deletions associated with superior 5-year PFS and OS –– RTOG 9402 ◦◦ 292 patients with either anaplastic astrocytoma or anaplastic oligodendroglioma –– Randomization: ◦◦ Postoperative radiation alone to 59.4 Gy in 33 fractions ◦◦ Postoperative procarbazine, CCNU, and vincristine x 4 cycles → RT –– Results: ◦◦ Codeleted tumors vs non-codeleted tumors, respectively • Median OS –– PCV + RT: 14.7 vs 2.6 years –– RT alone: 7.3 vs 2.7 years –– It is worth noting that the 2016 WHO classification of tumors of the central nervous system 2016 designates IDH mutant codeleted tumors as molecular oligodendroglioma even if there are histologic findings of astrocytoma. Hoang-Xuan K, Capelle L, et al. Temozolomide as initial treatment for adults with low-grade oligodendrogliomas or oligoastrocytomas and correlation with chromosome 1p deletions. J Clin Oncol. 2004; 22:3133–8. Bucker JC, Ballman KV, et  al. Diagnostic and prognostic significance of 1p and 19q deletions in patients (pts) with low-grade oligodendroglioma and astrocytoma. J Clin Oncol. 2005; 23:1145. Cairncross G, et  al. Phase III trial of chemoradiotherapy for anaplastic oligodendroglioma: long-term results of RTOG 9402. J Clin Oncol. 2013;31(3):337–43. 16. The correct answer is A. MGMT • DNA repair enzyme that repairs damage induced by alkylating agents, such as temozolomide • Methylation of the MGMT promoters → epigenetic silencing → decreased MGMT expression → less DNA damage repair • Hegi et al. –– Determined methylation status of GBM patients randomized to RT +/− temozolomide and found MGMT promoter methylation to be associated with improved survival

1  Central Nervous System Tumors: Adult and Pediatric

25

Hegi ME, et al. MGMT gene silencing and benefit from temozolomide in glioblastoma. N Engl J Med. 2005; 352:997–1003. 10. The correct answer is D.  Seizure is the most common presenting symptom, occurring in approximately two-thirds of patients. Brown PD, Shaw EG, van den Bendt MJ. Lowgrade gliomas. In: Gunderson LL, Tepper JE, editors, Clinical radiation oncology. 3rd ed. Philadelphia: Elsevier Saunders; 2012. p. 443–59. 17. The correct answer is D.  Seizure is the most common presenting symptom, occurring in approximately two-thirds of patients. Brown PD, Shaw EG, van den Bendt MJ. Low-grade gliomas. In: Gunderson LL, Tepper JE, editors, Clinical radiation oncology. 3rd ed. Philadelphia: Elsevier Saunders; 2012. p. 443–59. 18. The correct answer is C. Pignatti et al. prognostic scoring system of low-grade glioma • Derived from databases EORTC trials 22,844 and 22,845 • The following were found by Cox regression to be negative risk factors: –– Age 40 years or more –– Astrocytoma histology –– Largest diameter of the tumor ≥6 cm –– Tumor crossing the midline –– Presence of neurological deficit before surgery • Risk stratification –– Low-risk ◦◦ 0–2 risk factors ◦◦ Median survival of 7.7 years –– High-risk ◦◦ ≥3 risk factors ◦◦ Median survival of 3.2 years Pignatti F, van den Bent M, et al. Prognostic factors for survival in adult patients with cerebral low-grade glioma. J Clin Oncol. 2002;20:2076–84. 19. The correct answer is B. EORTC 22844 aka “believer’s trial” • Patients with low-grade glioma –– Only 25% underwent gross total resection prior to radiation. • Randomized: –– Immediate postoperative RT to 45 Gy –– Immediate postoperative RT to 59.4 Gy • RT CTV expansions: –– CTV + 2 cm: 45 Gy –– CTV + 1 cm: 54 Gy –– CTV + 0 cm: 59 Gy • Results: –– Dose escalation did not improve OS or PFS.

26

B. Murphy et al.

• Side note: –– Named “believer’s trial” as those who enrolled patients believed: ◦◦ Radiation had an impact in low-grade glioma. ◦◦ Increased dose would lead to improved clinical outcomes. –– In contrast, the “nonbeliever’s trial” in question 20, “nonbelief” was that radiation had no clinically significant benefit though this was trial of early vs delayed radiation therapy. Karim AB, et al. A randomized trial on dose-response in radiation therapy of low-­grade cerebral glioma: European Organization for Research and Treatment of Cancer (EORTC) Study 22844. Int J Radiat Oncol Biol Phys. 1996;36(3):549–56. 20. The correct answer is C. EORTC 22845 • Low-grade glioma patients • Compared early RT versus RT at recurrence of 54 Gy • Results: –– PFS improved in the early radiotherapy arm: 5.3 years vs 3.4 years. –– No difference in OS: 7.4 early RT vs 7.2 years RT at recurrence. –– Better 1-year seizure control in the early radiotherapy group: 25% vs 41%, p = 0.03. Van de Bent MJ, et al. Long-term efficacy of early versus delayed radiotherapy for low-grade astrocytoma and oligodendroglioma in adults: the EORTC 22845 randomised trial. Lancet. 2005;355:985–90. 21. The correct answer is C. The EORTC 26951 trial randomized 368 patients with anaplastic oligodendroglioma after surgery and radiation to either observation or adjuvant PCV chemotherapy (procarbazine, lomustine, vincristine). The initial published 5-year results in 2006 showed only a progression-free survival benefit (1.9 years vs 1.1 years). 1p19q deletion was subsequently evaluated, and those with 1p19q deletion had a 5-year overall survival of 74% versus 30% without the deletion. With 12 years of follow-up, the addition of PCV chemotherapy not only improves progression-free survival but also improves overall survival. Median survival is significantly prolonged (42.3  months vs 30.6 months) in those that received adjuvant PCV. In those with 1p19q deletion that received adjuvant PCV, median survival has not been reached. Van den Bent MJ, et al. First analysis of EORTC 26951, a randomised phase III study of adjuvant PCV chemotherapy in patients with highly anaplastic oligodendroglioma. J Clin Oncol. 2006;24(18):2715–22. Van den Bent MJ et al. Long-term follow-up results of EORTC 26951 a randomised phase III study of adjuvant. PCV chemotherapy in anaplastic oligodendroglial tumors (AOD). J Clin Oncol. 2012;(Suppl; abstr 2). 22. The correct answer is C. Magnetic resonance spectroscopy is a method to monitor biochemical changes in the brain. The most commonly used brain metabolites in MRS are choline, a measure of increased cell turnover, which is elevated

1  Central Nervous System Tumors: Adult and Pediatric

27

in tumors and inflammatory processes; creatine, a measure of energy stores, decreased in tumors; and N-acetylaspartate (NAA), a neuronal marker which decreases in processes that adversely affect neuron integrity. The MR spectrum shows various peaks of these metabolites, displayed from left to right—choline, creatine, and NAA. Hunter’s angle is a line drawn along the peaks of the white matter metabolites and is generally downward in brain tumors (denoting an increased choline to creatine ratio). Radiation necrosis on MRS is usually denoted by low levels of NAA, creatine, and choline. Nelson SJ, et  al. Characterization of untreated gliomas by magnetic resonance imaging. Neuroimag Clin. 2002;12:599–613. Graves EE, et al. Serial proton MR spectroscopic imaging of recurrent malignant gliomas after gamma knife radiosurgery. AJNR. 2001;22:613–24. 23. The correct answer is A. To attempt to determine an optimal radiation dose for malignant gliomas, ECOG/RTOG performed a four-arm study, comparing three experimental arms:(1) 60 Gy WBRT +10 Gy partial brain boost, (2) 60 Gy WBRT + BCNU, and (3) 60 Gy WBRT + CCNU + dacarbazine (DTIC) against a standard 60 Gy WBRT arm. There was no survival benefit seen with the 10 Gy boost. Age was found to be the most important prognostic factor, with 18-month survival 64% in those 60 years. Among the 40–60 year-old group, the addition of BCNU did improve survival compared to 60  Gy WBRT alone. Not unexpectedly, the ­combination of CCNU + DTIC was more toxic than BCNU, producing a higher incidence of thrombocytopenia. Chang CH, et al. Comparison of postoperative radiotherapy and combined postoperative radiotherapy and chemotherapy in the multidisciplinary management of malignant gliomas. A Joint Radiation Therapy Oncology Group and Eastern Cooperative Oncology Group Study. Cancer. 1983;(6):997–1007. 24. The correct answer is D. Lee et al. reported on the patterns of failure following high-dose 3-D conformal radiotherapy of high-grade astrocytomas. All patients were treated to either 70 or 80 Gy. An “in-field” recurrence was defined as 80% or more of the volume of the recurrence lying within the 95% isodose region. It was termed as “outside” when 20% or less of the volume of the recurrence was inside the 95% isodose region. Of the 36 evaluable patients, 89% failed “in-­ field,” with only 1 failing outside the high-dose region. Despite the dose escalation, there was no improvement in overall survival. No dose-limiting toxicity was seen. Lee SW, et al. Patterns of failure following high-dose 3-D conformal radiotherapy for high-grade astrocytomas: a quantitative dosimetric study. Int J Radiat Oncol Biol Phys. 1999;43:79–88. 25. The correct answer is C. RTOG 90–05 sought to define optimal doses for radiosurgery in patients with recurrence malignant brain tumors, including both gliomas and metastases. Patients were stratified by tumor volume, 7y) and progression-free survival. An update was reported at the 2012 ASCO meeting. With 11.3  years of follow-up, the 1p/19q deleted patients showed a median survival of 14.7  years with PCV versus 7.3  years without (p = 0.03). Cairncross JG, et al. Phase III trial of chemotherapy plus radiotherapy alone for pure and mixed anaplastic oligodendroglioma: intergroup radiation therapy oncology group trial 9402. J Clin Oncol. 2006;24(18):2707–14. Carincross JG, et al. Chemotherapy plus radiotherapy (CT-RT) versus RT alone for patients with anaplastic oligodendroglioma: long-term results of the RTOG 94–02 phase III study. J Clin Oncol. 2012 (Suppl; abstr 2008b). 55. The correct answer is D.  The WHO (World Health Organization) published revised grading for meningiomas in 2000, based on clinicopathological series from the Mayo Clinic. The revision of the histological grading of meningiomas resulted in upgrading of previously benign meningiomas to the atypical category. In contrast, the criteria for grading anaplastic or malignant meningiomas became stricter. In the new classification, 20 mitoses per 10 hpf are required for classification of an anaplastic meningioma. Atypical meningioma was classified by at least 4 mitoses in 10 hpf or 3 of the following criteria: increased cellularity, high nuclear-to-cytoplasm ratios, prominent nucleoli, sheet-like

1  Central Nervous System Tumors: Adult and Pediatric

35

growth, and necrosis. The most notable change from the 2007 WHO scheme was that brain invasive meningiomas are classified as WHO grade II based on similar recurrence and mortality rates to those of atypical meningiomas. Louis DN, Scheithauer BW, Budka H, Von Deimling A, Kepes JJ. Meningiomas. In: Kleihues P, Cavenee WK, editors. Pathology and genetics of tumors of the nervous system. Lyon: IARC Press; 2000. p.  176–84. Perry A, Louis DN, Scheithauer BW, Budka H, Von Deimling, A.  Meningiomas. In: Louis DN, OhgakiH, Wiestler OD, Cavenee WK, editors. WHO classification of tumours of the central nervous system. Lyon: IARC Press; 2007. p. 164–72. 56. The correct answer is A. Simpson grading describes the extent of resection of meningiomas: grade I, complete removal including resection of underlying bone and associated dura; grade II, complete removal and coagulation of dural attachment; grade III, complete removal without resection of dura or coagulation; and grade IV, subtotal resection. With increasing Simpson grade, there is increased local recurrence. Brooker classification describes the degree of heterotopic bone formation. Bloom-Richardson is the grading system used for breast carcinomas. Kadish staging is used for esthesioneuroblastoma. Simpson D.  The recurrence of intracranial meningiomas after surgical treatment. J Neurol Neurosurg Psychiatr. 1957;20(1):22–39. 57. The correct answer is C. The pineal gland is the most common site for CNS germ cell tumors, followed by the suprasellar region. The most common histology is germinoma. 15–30% of pineal tumors originate in pinealocytes and fall into one of the four following types: pineocytomas, mixed pineal parenchymal tumors, pineal parenchymal tumors with intermediate differentiation, and pineoblastomas. In patients with bilateral retinoblastomas, a pineoblastoma is called trilateral retinoblastoma. Merchant TE. Central nervous system tumors in children. In: Gunderson LL, Tepper JE, editors. Clinical radiation oncology. 3rd ed. Philadelphia: Elsevier Saunders; 2012. p. 1409–23. 58. The correct answer is B. Parinaud’s syndrome, associated with pineoblastoma, results from pressure on the superior colliculus in the midbrain. The syndrome consists of vertical gaze palsy, light-near dissociation of the pupils, and convergence retraction nystagmus. Eaton-Lambert and limbic encephalitis are paraneoplastic syndromes seen with small-cell lung cancer. Merchant TE. Central nervous system tumors in children. In: Gunderson LL, Tepper JE, editors. Clinical radiation oncology. 3rd ed. Philadelphia: Elsevier Saunders; 2012. p. 1409–23. 59. The correct answer is D. Of the listed tumors found in the pineal gland, pineocytomas have the least propensity for craniospinal dissemination. Treatment usually consists of at least 50  Gy to the primary tumor alone. Merchant TE. Central nervous system tumors in children. In: Gunderson LL, Tepper JE, editors. Clinical radiation oncology. 3rd ed. Philadelphia: Elsevier Saunders; 2012. p. 1409–23.

36

B. Murphy et al.

60. The correct answer is B.  While hemangioblastomas are characterized by mutations in the VHL tumor suppressor gene on chromosome 3p, hemangiopericytomas are not characterized by any specific patterns of genetic change. Hemangioblastomas tend to be slow growing with indolent behavior, whereas hemangiopericytomas tend to be more locally aggressive, recurring even after complete surgical resection. Surgical resection is the mainstay of therapy for both tumors. Radiotherapy can be given after subtotal resection of hemangioblastomas, while radiation is recommended even after complete resection of hemangiopericytomas, given their propensity for LR.  Stieber VW, Siker ML. Spinal cord tumors. In: Gunderson LL, Tepper JE, editors. Clinical radiation oncology. 3rd ed. Philadelphia: Elsevier Saunders; 2012. p. 511–28. 61. The correct answer is C. Craniopharyngiomas are benign suprasellar neoplasms derived from remnants of Rathke’s pouch. There is a bimodal incidence: children from 5 to10 years then adults from 55 to 65 years. Radiographically, calcification is frequently seen. There are solid nodule and cystic components, which may be filled with cholesterol-laden fluid (“crankcase oil”). The development of diabetes insipidus is a frequent complication from surgery. Merchant TE. Central nervous system tumors in children. In: Gunderson LL, Tepper JE, editors. Clinical radiation oncology. 3rd ed. Philadelphia: Elsevier Saunders; 2012. p. 1409–23. 62. The correct answer is A. Approximately 30% of pituitary adenomas are non-­ secreting. Of the secreting tumors, prolactinomas are the most common, followed by GH-secreting tumors. Thapar K et al. Pituitary tumors. Cancer of the nervous system. Cambridge: Blackwell Scientific; 1997. p. 363. Suh JH, Chao ST, Weil RJ. Pituitary tumors. In: Gunderson LL, Tepper JE, editors. Clinical radiation oncology. 3rd ed. Philadelphia: Elsevier Saunders; 2012. p. 493–509. 63. The correct answer is A.  The posterior lobe of the pituitary, the neurohypophysis, secretes ADH and oxytocin. The anterior lobe of the pituitary, the adenohypophysis, secretes prolactin, ACTH, FSH, LH, GH, and TSH.  Suh JH, Chao ST, Weil RJ. Pituitary tumors. In: Gunderson LL, Tepper JE, editors. Clinical radiation oncology. 3rd ed. Philadelphia: Elsevier Saunders; 2012. p. 493–509. 64. The correct answer is B.  There is no effective medical therapy for a TSH-­ secreting pituitary adenoma. Prolactinomas can be treated with bromocriptine. ACTH-secreting adenomas can be treated with cyproheptadine. GH-secreting adenomas can be treated with either octreotide (long-acting somatostatin analogue) or pegvisomant (GH receptor antagonist). Suh JH, Chao ST, Wei RJ. Pituitary tumors. In: Gunderson LL, Tepper JE, editors. Clinical radiation oncology. 3rd ed. Philadelphia: Elsevier Saunders; 2012. p. 493–509.

1  Central Nervous System Tumors: Adult and Pediatric

37

65. The correct answer is D. After irradiation of the hypothalamic-pituitary axis, growth hormone deficiency is most commonly seen, followed by deficiency of adrenocorticotropin, gonadotropin, and thyroid-stimulating hormone. Robinson IC, et al. Differential radiosensitivity of hypothalamic-pituitary function in the young adult rat. J Endocrinol. 2001;169:519–26. Merchant TE, et al. Early neuro-otologic effects of three-dimensional irradiation in children with primary brain tumors. Int J Radiat Oncol Biol Phys. 2004;58:1194–207. 66. The correct answer is C. The cavernous sinus is located lateral to the pituitary and transmits the internal carotid artery, CN III, CN IV, CN VI (six), and the V1 (one) and V2 branches of the trigeminal nerve. The nerves transmitted from the cranial to caudal direction are CN III, CN IV, CN VI (six), CN V1 (V one), and CN V2. CN V2, the maxillary branch of the trigeminal nerve, is the most caudal structure in the cavernous sinus. 67. The correct answer is A. Of the primary ocular tumors, melanoma is the most common. Retinoblastoma is the second most common. Primary orbital lymphomas are rare and account for less than 1% of diagnosed lymphomas. Greven KM, Greven CM. Orbital, ocular, and optic nerve tumors. In: Gunderson LL, Tepper JE, editors. Clinical radiation oncology. 3rd ed. Philadelphia: Elsevier Saunders; 2012. p. 529–42. 68. The correct answer is D.  Of the primary ocular tumors, optic nerve sheath meningiomas occur more frequently in women than men, with a typical age of onset of 40 years. Retinoblastoma and melanoma have no gender predilection. Orbital lymphomas occur more frequently in men. Greven KM, Greven CM. Orbital, ocular, and optic nerve tumors. In: Gunderson LL, Tepper JE, editors. Clinical radiation oncology. 3rd ed. Philadelphia: Elsevier Saunders; 2012. p. 529–42. 69. The correct answer is C. The most common tumor to metastasize to the orbit, as well as to the choroid, is breast cancer. Shields JA et al. Survey of 1264 patients with orbital tumors and simulating lesions: the 2002 Montgomery Lecture, part 1. Ophthalmology. 2004; 111:997–1008. Shields CL, et  al. Survey of 520 eyes with uveal metastases. Ophthalmology. 1997;104:1265. 70. The correct answer is B. The Collaborative Ocular Melanoma Group (COMS) study established a standard size classification for ocular melanomas: small, 1–3 mm thick and 16  mm in the greatest dimension. The Collaborative Ophthalmology Study Group. The collaborative ocular melanoma study randomized trial of iodine-125 brachytherapy for choroidal melanoma, III: initial mortality findings. COMS report No. 18. Arch Ophthalmol. 2001;119:969.

38

B. Murphy et al.

71. The correct answer is D. COMS study for large choroidal melanomas randomized patients to receive or not preoperative external beam radiotherapy to 20 Gy, prior to planned enucleation. The addition of EBRT did not improve overall survival nor local control. Collaborative Ocular Melanoma Group. The collaborative ocular melanoma study randomized trial or pre-enucleation radiation of large choroidal melanoma, II: initial mortality findings. COMS report No. 10. Am J Ophthalmol. 1998; 125:779. 72. The correct answer is C. Acoustic neuromas, commonly seen in neurofibromatosis 2, most commonly affect the vestibular division of CN VIII. Chan MD, Rogers CL, Anderson BM, Khuntia D. benign brain tumors: meningiomas and vestibular schwannomas. In: Gunderson LL, Tepper JE, editors. Clinical radiation oncology. 3rd ed. Philadelphia: Elsevier Saunders; 2012. p. 473–91. 73. The correct answer is C.  Of the listed subtypes, germinoma is not a type of embryonal CNS tumor. Embryonal CNS tumors are of two major groups: ­primitive neuroectodermal tumor (PNET)—medulloblastoma and pineoblastoma—and atypical teratoid/rhabdoid tumors (AT/RT). Germinomas are germ cell tumors. Merchant TE. Nervous system tumors in children. In: Gunderson LL, Tepper JE, editors. Clinical radiation oncology. 3rd ed. Philadelphia: Elsevier Saunders; 2012. p. 1409–23. 74. The correct answer is B. Embryonal CNS tumors have a propensity for neuraxis dissemination. As such, craniospinal irradiation is the mainstay of treatment of these tumors. Merchant TE. Central nervous system tumors in children. In: Gunderson LL, Tepper JE, editors. Clinical radiation oncology. 3rd ed. Philadelphia: Elsevier Saunders; 2012. p. 1409–23. 75. The correct answer is A. At least four molecular subtypes of medulloblastoma and their differential outcomes are known. The Wnt pathway, associated with monosomy 6 and occurring in all age groups, has the best prognosis. The SHh pathway is associated with desmoplastic histology, occurring in infants and adults, and has a good prognosis. The c-MYC subtype occurs in children 3–10 years and has the worst prognosis. Other subtypes occur in all age groups and have a fair prognosis. Monje M, et al. Hedgehogs, flies, wnts, and MYCs: the time has come for many things in medulloblastoma. J Clin Oncol. 2011;29(11):1395–98. Cho Y-J, et al. Integrative genomic analysis of medulloblastoma identifies a molecular subgroup that drives poor clinical outcome. J Clin Oncol. 2011;29:1424–30. Northcott PA, et  al. Medulloblastoma comprises four distinct molecular variants. J Clin Oncol. 2011;29:1408–14.

1  Central Nervous System Tumors: Adult and Pediatric

39

76. The correct answer is B.  The borders of the posterior fossa are superior-­ tentorium cerebelli, inferior-occipital bone, anterior-clivus, posterior-­calvarium, lateral-temporal, parietal, and occipital bones. Merchant TE. Central nervous system tumors in children. In: Gunderson LL, Tepper JE, editors. Clinical radiation oncology. 3rd ed. Philadelphia: Elsevier Saunders; 2012. p. 1409–23. 77. The correct answer is C. The previous approach to standard-risk medulloblastoma was 36 Gy to the craniospinal axis, followed by a boost to the posterior fossa to a total of 54 Gy. POG 8631/CCG 923 compared 36 Gy to the craniospinal axis to 23.4 Gy and showed a higher risk of relapse with the lower craniospinal dose. However, CCG-9892 subsequently showed that the two doses were equivalent if the 23.4 Gy craniospinal dose was given in combination with multiagent chemotherapy. Thomas PR, et al. Low-stage medulloblastoma: final analysis of standard-dose with reduced-dose neuraxis irradiation. J Clin Oncol. 2000;18:3004–11. Packer RJ, et  al. Treatment of children with medulloblastoma with reduced-dose craniospinal radiation therapy and adjuvant chemotherapy. J Clin Oncol. 1999;17(7):2127–36. 78. The correct answer is A. A St. Jude Children’s Hospital prospective trial sought to reduce the volume treated with radiotherapy in order to reduce neurocognitive sequelae. In this prospective trial, 86 patients aged 3–21, with standard-risk medulloblastoma were treated with craniospinal irradiation (23.4 Gy), followed by irradiation to a total of 36 Gy to the posterior fossa and followed by a tumor bed +2 cm boost to a total of 55.8 Gy. Chemotherapy (four cycles of cyclophosphamide, cisplatin, vincristine) began 6 weeks after radiation. Three-year posterior fossa failure was 6.3%, and the reduction in radiotherapy targeting volume resulted in a 13% decrease in volume of posterior fossa receiving doses >55 Gy. The dose reductions to the temporal lobes, cochleae, and hypothalamus were statistically significant. Merchant TE, et  al. Multi-institution prospective trial of reduced-dose craniospinal irradiation (23.4  Gy) followed by conformal posterior fossa (36 Gy) and primary site irradiation (55.8 Gy) and dose-intensive chemotherapy for average-risk medulloblastoma. Int J Radiat Oncol Biol Phys. 2008;70(3):782–7. 79. The correct answer is C. Due to neurocognitive sequelae, there has been a push to delay radiotherapy use in children 3 mm. 24. Which is the most common variant of melanoma? A. Superficial spreading B. Nodular melanoma C. Lentigo maligna melanoma D. Acral lentiginous 25. Which variant of melanoma is most likely to be found under the nail of the thumb? A. Superficial spreading B. Mucosal melanoma C. Lentigo maligna melanoma D. Acral lentiginous 26. A nodular melanoma is found on a 60-year-old woman’s right cheek. It is ulcerated and 1.5 mm thick. There are no regional lymph nodes. What is the correct 8th edition AJCC stage? A. T1aN0 B. T1bN0 C. T2aN0 D. T2bN0 27. Melanoma that invades but does not fill the papillary dermis is Clark level: A. II B. III C. IV D. V

52

R. F. Thompson et al.

28. Which one of the following patients is least likely to undergo sentinel lymph node biopsy? A. Ulcerated 1-mm-thick melanoma B. 1-mm-thick, Clark level IV melanoma C. 1-mm-thick melanoma with satellitosis D. Non-ulcerated 1-mm-thick melanoma 29. Which is true regarding the management of thin stage I primary cutaneous melanoma lesions? A. Radiation therapy does not typically play a role. B. Wide local excision of lesions 1 mm thick should include 1 cm of clear margins. D. Adjuvant radiation after wide local excision is important because local recurrence with excision alone is >20%. 30. All are true regarding nodal treatment for melanoma, except: A. No randomized trials have shown a survival benefit for elective nodal dissection at the time of initial surgery. B. Sentinel lymph node biopsy can identify those patients that will benefit from nodal dissection. C. Completion lymph node dissection in patients with sentinel lymph node metastasis has been shown to improve overall survival. D. Elective regional irradiation of head and neck melanoma has been shown to yield almost 90% regional control at 10 years. 31. Which one of the following scenarios is least likely to require adjuvant nodal irradiation? A. Single 1.5 cm lymph node found on elective axillary lymph node dissection, complicated by minor lymphedema B. Four lymph nodes found on therapeutic inguinal lymph node dissection, largest measuring 4 cm C. Several lymph nodes with extracapsular extension found on therapeutic cervical lymph node dissection D. One 2.5 cm lymph node with extracapsular extension found on repeat axillary lymph node dissection 32. All of the following are true regarding talimogene laherparepvec (also known as Imlygic), except: A. Imlygic is effective for early-stage cutaneous melanoma. B. It is the first FDA-approved oncolytic viral therapy in the United States. C. The exact mechanism of response is unknown. D. Imlygic is delivered intratumorally, but can have distant (systemic) antitumor effects.

2  Skin Cancer and Uveal Melanoma

53

33. Which of the following clinical scenarios represents an example of the abscopal effect? A. Metastatic melanoma exhibiting a systemic partial response to dual checkpoint inhibitor therapy B. Metastatic melanoma exhibiting a systemic partial response to checkpoint inhibitor monotherapy, treated with SRS for a new single brain metastasis C. Metastatic melanoma exhibiting a complete response after palliative chest wall irradiation for oligoprogression of three sites (chest wall, axilla, lung) on single-agent checkpoint blockade D. Metastatic melanoma exhibiting a complete response after initiation of dual checkpoint inhibitor therapy for systemic progression despite BRAF/MEK inhibitor therapy 34. Which of the following represents the correct pairing of therapy and clinical indication? A. Vismodegib – metastatic basal cell carcinoma B. Talimogene laherparepvec – metastatic Merkel cell carcinoma C. PUVA – unresectable squamous cell carcinoma D. Electron radiotherapy – unresectable nasal mucosal melanoma 35. All of the following are true regarding a randomized trial comparing dacarbazine +/− ipilimumab reported by Robert et al., except: A. The addition of ipilimumab improved overall survival. B. Median survival was improved 2 months with the addition of ipilimumab. C. The most common reason for ipilimumab discontinuation was gastrointestinal toxicity. D. Elevation of the liver enzymes was the most frequently observed bowel toxicity. 36. In a prospective single-arm study of the oral BRAF inhibitor, vemurafenib, all of the following are true, except: A. Vemurafenib is an inhibitor of the V600E BRAF mutation. B. Patients with all sites of metastases were included. C. There was an >80% response rate in those with the V600E mutation. D. Disease regression among responders was transient, lasting between 2 and 18 months. 37. What radiation fractionation was used in a prospective series from MD Anderson Cancer Center (Ang et  al.) using hypofractionated postoperative radiotherapy for cutaneous melanoma of the head and neck region? A. 30 Gy in 6 fractions B. 24 Gy in 6 fractions C. 30 Gy in 5 fractions D. 24 Gy in 4 fractions

54

R. F. Thompson et al.

38. What is the approximate 5-year locoregional control rate according to a prospective series from MD Anderson Cancer Center (Ang et al.) using hypofractionated postoperative radiotherapy for cutaneous melanoma of the head and neck region? A. 55% B. 65% C. 75% D. 85% 39. The Tasman Radiation Oncology Group randomized trial of postoperative radiotherapy for melanoma at high risk for local relapse employed which radiotherapy dose fractionation: A. 30 Gy in 5 fractions B. 30 Gy in 6 fractions C. 48 Gy in 12 fractions D. 48 Gy in 20 fractions 40. Which of the following pathologic scenarios would not have been eligible for the Tasman Radiation Oncology Group randomized trial of postoperative radiotherapy for melanoma at high risk for local relapse? A. One positive cervical lymph node B. Two positive parotid lymph nodes C. Three positive groin nodes D. One positive groin node with extracapsular extension 41. Which of the following is correct regarding the outcome of the Tasman Radiation Oncology Group randomized trial of postoperative radiotherapy for melanoma at high risk for local relapse? A. Improved local control and improved survival with post-op RT B. No improved local control and no improved survival with post-op RT C. Improved local control but no improved survival with post-op RT D. No improved local control but improved survival with post-op RT 42. To provide a ≥95% chance of covering microscopic disease with radiation for non-melanomatous skin cancer, what CTV margin is required for SCC and BCC, respectively? A. 6 mm, 8 mm B. 10 mm, 11 mm C. 13 mm, 14 mm D. 15 mm, 15 mm 43. Which of the following targeted agents has shown therapeutic benefit in advanced cutaneous basal cell carcinoma not amenable to surgical intervention? A. Cetuximab B. Vismodegib C. Panitumumab D. Erlotinib

2  Skin Cancer and Uveal Melanoma

55

44. Which of the following is the target of the targeted agent that has shown therapeutic benefit in advanced cutaneous basal cell carcinoma not amenable to surgical intervention? A. Smoothened (SMO) B. Epidermal growth factor receptor (EGFR) C. Sonic hedgehog (SHH) D. Wingless and Int-1 (WNT) 45. What is most common cutaneous malignancy in solid organ transplant receipts? A. Merkel cell carcinoma B. Basal cell carcinoma C. Melanoma D. Squamous cell carcinoma 46. A patient presents with cutaneous squamous cell carcinoma of the left malar cheeks who undergoes excision with negative margins. There was presence of extensive perineural invasion. When delineating the CTV for perineural invasion for this particular patient, which nerve located in the pterygopalatine fossa is included? A. Infraorbital nerve B. Maxillary nerve C. Mandibular nerve D. Inferior alveolar nerve 47. A patient presents with cutaneous squamous cell cancer of the lower lip who underwent with limited excision and found to have extensive perineural invasion. Further resection was deferred by the patient for cosmetic reasons and electing to undergo radiation therapy. When covering the nerve tracts associated with this site within the CTV, between which two structures is the nerve located after it exits the mandibular foramen? A. Masseter and buccinator muscles B. Temporalis and lateral pterygoid muscles C. Lateral and medial pterygoid muscles D. Parotid gland and masseter muscles 48. Per ASTRO 2019 Clinical Practice Guidelines, in which of the following cases is definitive radiation for cutaneous basal or squamous cell carcinoma not strongly recommended? A. An 88-year-old man with multiple comorbidities and high perioperative risk who presents with a 4.0 cm lesion of the left malar cheek B. A 46-year-old lady with a 1.0 cm lesion involving the right oral commissure C. A 60-year-old man with a 1.0 cm lesion involving the left medial canthus D. A 21-year-old man with bony jaw pain due to multiple cystic bony lesions who presents with 1.5 cm lesion of the left nasal ala

56

R. F. Thompson et al.

49. Per ASTRO 2019 Clinical Practice Guidelines, in which of the following cases is postoperative radiation for cutaneous basal or squamous cell carcinoma not strongly recommended? A. A 58-year-old lady with a 1.5 cm SCC of the scalp with positive margins after re-excision B. A 64-year-old man with a paresthesia of the right cheek who presents with a 1.0 cm lesion underneath his left orbit C. A 44-year-old man with a 3.9 cm lesion of the right posterior neck D. A 37-year-old man with history of liver transplantation with a 1.0 cm lesion superior to the left upper eyebrow and pathology noting desmoplasia 50. Per Brantsch et  al. who prospectively investigated risk factors for metastasis and local recurrence of SCC, what are the respective approximate risks of local recurrence if there is a presence of desmoplasia or tumor thickness >6.0 mm? A. 5%, 50% B. 25%, 30% C. 25%, 15% D. 50%, 5% 51. Upon planning definitive radiation therapy to an inoperable squamous cell carcinoma of the right temporal scalp with radiographically evident supraorbital nerve invasion, which of the following lacrimal gland constraints is a desired clinical? A. Dmax 5 mm C. Tumor location near optic nerve, maximal basal tumor diameter >11 mm D. Tumor location near optic nerve, tumor apical height >5 mm

58

R. F. Thompson et al.

59. All of the following are accepted proton beam doses and fractionation except: A. 60 GyE in 4 fractions B. 70 GyE in 5 fractions C. 50 GyE in 5 fractions D. 80CGE in 6 fractions 60. The COMS randomized trial compared I-125 plaque brachytherapy vs enucleation for small to medium choroidal melanomas. All of the following are true except: A. 10-year all-cause mortality in all subjects was 35%. B. Enucleation had superior all-cause mortality. C. No difference was seen in all-cause mortality or death with metastatic melanoma. D. 10-year death with metastatic melanoma in all subjects was 18% in the I-125 plaque arm and 17% in the enucleation arm. 61. Which of the following are all toxicities from eye plaque brachytherapy? A. Radiation retinopathy, ipsilateral eye vision loss, eyelash loss B. Optic neuropathy, glaucoma, cataract C. Cataract, radiation retinopathy, brain damage D. Scleral necrosis, contralateral eye vision loss, optic neuropathy

2  Skin Cancer and Uveal Melanoma

59

Answers and Rationale 1. The correct answer is C. • Basal cell and squamous cell carcinomas together constitute the most common malignancy in the United States. • Basal cell carcinomas –– Four times more common than squamous cell carcinomas. –– More indolent growth pattern. –– They rarely metastasize. –– Cure rates are high. –– Surgical resection is the primary modality of treatment, except for BCCs of the nose, ear, and lower eyelids, in which radiation may have better functional and cosmetic results. Jemal A et al. Cancer statistics 2005. CA Cancer J Clin. 2005; 55:10–30. 2. The correct answer is A. All of the listed syndromes are associated with increased incidence of skin carcinomas except ataxia-telangiectasia. Cleaver JE.  Defective repair replication of DNA in xeroderma pigmentosum. Nature. 1968; 218:652–6. Anderson DE et al. The nevoid basal cell carcinoma syndrome. Am J Hum Genet. 1967; 19:12–22. Lutzner MA. Epidermodysplasia verruciformis: an autosomal recessive disease characterized by viral warts and skin cancer. A model for viral oncogenesis. Bull Cancer (Paris). 1978; 65:169–82. 3. The correct answer is C. Basal cell nevus syndrome (Gorlin syndrome): • Inherited in an autosomal dominant pattern • Comprises a group of defects involving the skin, nervous system, eyes, endocrine glands, and bone • Physical symptoms of the syndrome –– Multiple nevoid basal cell carcinomas –– Broad nose –– Cleft palate –– Protruding brow –– Protruding jaw with odontogenic keratocysts (jaw cysts) –– Plantar/palmar pits –– Hypertelorism (wide-set eyes) • There is also an association with medulloblastoma. Gorlin RJ, Goltz RW. Multiple nevoid basal-cell epithelioma, jaw cysts and bifid rib: a syndrome. N Engl J Med. 1960;262:908–12. Morelli JG. Tumors of the skin. In: Kliegman RM, Behrman RE, Jenson HB, Stanton BF, editors. Nelson textbook of pediatrics. 19th ed. Philadelphia: Saunders Elsevier; 2011 chap 662.

60

R. F. Thompson et al.

4. The correct answer is A. • The most common variant of basal cell carcinoma is the nodular-ulcerative (rodent ulcer) variant. –– Presents as a papule with a central umbilication that grows into a central ulceration –– Can be confused with melanoma • Superficial basal cell carcinoma. –– Presents as red, scaly macules with indistinct margins –– Can be confused with psoriasis • Morpheaform basal cell carcinoma. • Presents as flat, indurated, ill-defined macules with a shiny surface that depresses as it grows into a plaque • Infiltrative basal cell carcinoma appears yellowish and blends into adjacent skin. Patterson JAK et al. Cancers of the skin. In: DeVita VT, Hellman S, Rosenberg SA, editors. Cancer principles and practice of oncology. 3rd ed. Philadelphia: JB Lippincott; 1989. p. 1469–98. 5. The correct answer is B. The 5th (CN V, trigeminal) and 7th (CN VII, facial) cranial nerves are most likely to be affected by a locally aggressive BCC. Veness MJ, Howle J. Cutaneous Carcinoma. In: Gunderson LL, Tepper JE, editors. Clinical radiation oncology. 4th ed. Philadelphia: Elsevier Saunders; 2016. p.763–776. 6. The correct answer is A. Merkel cell carcinoma: • Neoplasm that arises from neural crest cells. • Firm, painless, pink nodule or plaque in the head/neck or extremities. • Lymph nodes are involved at presentation in 20% of cases. • Relatively radiosensitive tumors. Morrison WH et al. The essential role of radiation therapy in securing locoregional control of Merkel cell carcinoma. Int J Radiat Oncol Biol Phys. 1990;19:583–91. Merkel F. Tastzellen und Tastkoerperchen bei den haushieren und beim Menschen. Arch Mikr Anat. 1875;11:636–52. 7. The correct answer is B. Lymph nodes are involved at presentation in 20% of cases. Please refer to Question 6. Morrison WH et al. The essential role of radiation therapy in securing locoregional control of Merkel cell carcinoma. Int J Radiat Oncol Biol Phys. 1990;19:583–91. Merkel F. Tastzellen und Tastkoerperchen bei den haushieren und beim Menschen. Arch Mikr Anat. 1875;11:636–52.

2  Skin Cancer and Uveal Melanoma

61

8. The correct answer is D. Morrison et al.: • 54 patients with Merkel cell carcinoma at the MD Anderson Cancer Center • Treatment –– Excision –– Postoperative radiotherapy ◦◦ 60 Gy to the primary ◦◦ 51 Gy to the primary echelon nodes ◦◦ 50 Gy to the supraclavicular nodes • Results –– Overall local failure – 35% –– Overall regional failure – 67% –– 91% relapsed during the first year ◦◦ Median time to relapse – 4.9 months –– Predominant pattern of failure – distant metastasis –– Lymph node-positive patients ◦◦ mOS of 13 months compared to 40 months in lymph node-negative patients Morrison WH et al. The essential role of radiation therapy in securing locoregional control of Merkel cell carcinoma. Int J Radiat Oncol Biol Phys. 1990;19:583–91. 9. The correct answer is A. • Sebaceous carcinoma arises most commonly in the upper eyelid. • Second most common eyelid neoplasm (most common – BCC). • Second most lethal neoplasm (most lethal – melanoma). • Present slow-growing, non-tender nodules. • Behave more aggressively than squamous cell carcinoma. Civatte J et al. Adnexal skin carcinomas. In: Andrade R, Gumport SL, Popkin GL, et al. editors.: Cancer of the skin, vol 2. Philadelphia: WB Saunders; 1976. p.  1045–68. Rulon DB et  al. Cutaneous sebaceous neoplasms. Cancer. 1974;33:82–102. Batsakis JG et al. Sebaceous cell lesions of the head and neck. Arch Otolaryngol. 1972;95:151–7. Rao NA et al. Sebaceous carcinomas of the ocular adnexa: a clinicopathologic study of 104 cases, with five-year follow-up data. Hum Pathol. 1982;13:113–22. 10. The correct answer is D. A Marjolin’s ulcer refers to an aggressive, ulcerating squamous cell carcinoma that arises in an area of prior trauma, chronic inflammation, and, most commonly, a burn. Chong AJ et al. Images in clinical medicine. Marjolin’s ulcer. N Engl J Med. 2005;352:e9.

62

R. F. Thompson et al.

11. The correct answer is C. Nevus of Ota (congenital melanosis bulbi, oculodermal melanocytosis): • Blue hyperpigmentation of the face usually noted unilaterally. • Results from entrapment of melanocytes in the upper third of the dermis • Involves the first two branches of the trigeminal nerves. • The sclera is involved in 2/3 of cases. • Women are 5× more likely to develop nevus of Ota than men. • More common in Asians (rare in whites). Rapini RP et  al. Dermatology: 2-volume set. St. Louis: Mosby. p.  1720–2. Chan HH, Kono T. Nevus of Ota: clinical aspects and management. Skinmed. 2(2):89–96. 12. The correct answer is C. • Stage I and II patients represent a remarkably heterogeneous group with a 3–55% chance of disease progression and death 5 years from diagnosis. • At least one gene expression profiling test has been validated in two independent primary cutaneous melanoma cohorts, with the ability to improve prediction of metastatic potential for early-stage melanoma. –– Cox regression analysis indeed indicates that the signature is an independent predictor of metastasis risk in the cohorts studied. • According to the MSLT-I trial in patients with clinically lymph node-­ negative disease: –– 16% of patients were found to have nodal metastasis on sentinel lymph node biopsy. Gerami P, et al. Development of a Prognostic Genetic Signature to Predict the Metastatic Risk Associated with Cutaneous Melanoma. Clin Cancer Res. 2015;21(1):175–83. Morton DL, et  al. Final Trial Report of Sentinel-Node Biopsy versus Nodal Observation in Melanoma. N Engl J Med. 2014;370(7):599–604.5 cm squamous cell carcinoma of the cheek is staged as T2, which includes tumors >2 cm but ≤5 cm. Edge SB, Byrd DR, Compton CC, et al., editors. Cutaneous squamous cell carcinoma and other cutaneous carcinomas. AJCC cancer staging handbook. 7th ed. New York: Springer; 2010. 13. The correct answer is A. • This small cancer exhibits two high-risk factors according to AJCC 7: –– Poorly differentiated histology –– Location on non-hair bearing lip –– (Previously stage T2 - ‘any size with ≥2 high risk features’) • AJCC 8th edition simply defines T1 v. T2 by size criteria alone: –– T1 90%. ◦◦ Size of the primary lesion is the major determinant of local control. • For tumors of the same size, basal cell carcinomas have higher control rates. • Late complications of radiation include fibrosis and atrophy. Fitzpatrick PJ et al. Basal and squamous cell carcinoma of the eyelids and their treatment by radiotherapy. Int J Radiat Oncol Biol Phys. 1984;10:449–54. Petrovich Z et al. Carcinoma of the lip and selected sites of head and neck skin. A clinical study of 896 patients. Radiother Oncol. 1987;8:11–7. 15. The correct answer is D. • Elective nodal irradiation is not a component of therapy for squamous or basal cell carcinomas, given their low incidence of nodal involvement. –– Except in cases of recurrent carcinoma after previous surgery • Indications for adjuvant nodal radiation: –– Lymph node metastases >3 cm –– Extracapsular extension on lymph node dissection Veness MJ, Howle J. Cutaneous Carcinoma. In: Gunderson LL, Tepper JE, editors. Clinical radiation oncology. 4th ed. Philadelphia: Elsevier Saunders; 2016. p. 763–76. 16. The correct answer is D. In previously untreated T4 skin carcinomas treated with 60–75 Gy over 6–8 weeks, 5-year local control was 67%. Those with bony invasion or perineural spread had reduced local control. Lee WR et al. Radical radiotherapy for T4 carcinoma of the skin of the head and neck: a multivariate analysis. Head Neck. 1993;15:32–324. 17. The correct answer is C. • Smoking is not known to be a risk factor for melanoma. • Risk factors of cutaneous melanoma include: –– History of significant UV exposure ◦◦ Sunburns ◦◦ Indoor tanning bed use ◦◦ Significant occupational or recreational exposure –– Fair skin

64

R. F. Thompson et al.

–– Red hair –– Green or blue eyes –– Large number of moles –– Freckles Ballo MT, Burmeister BH.  Malignant melanoma. In: Gunderson LL, Tepper JE, editors. Clinical radiation oncology. 4th ed. Philadelphia: Elsevier Saunders; 2016. p. 777–87. 18. The correct answer is A. Of the listed subtypes, superficial type is not a subtype of Merkel cell carcinoma. The trabecular subtype is the most common subtype. Veness MJ, Howle J. Cutaneous Carcinoma. In: Gunderson LL, Tepper JE, editors. Clinical radiation oncology. 4th ed. Philadelphia: Elsevier Saunders; 2016. p.763–76. 19. The correct answer is D. Merkel cell tumors are neuroendocrine tumors with immunohistochemistry of: • Positive neuron-specific enolase • Negative S-100 (melanoma is S-100 positive) • Negative leukocyte antigen (positive in lymphoma) Veness MJ, Howle J. Cutaneous Carcinoma. In: Gunderson LL, Tepper JE, editors. Clinical radiation oncology. 4th ed. Philadelphia: Elsevier Saunders; 2016. p. 763–76. 20. The correct answer is A.  Melanoma stains positive for both S-100 and HMB- 45. Nonaka D et al. Differential expression of S100 protein subtypes in malignant melanoma, and benign and malignant peripheral nerve sheath tumors. J Cutan Pathol. 2008;35(11);1014–19. Gown AM et al. Monoclonal antibodies specific for melanocytic tumors distinguishes subpopulations of melanocytes. Am J Pathol. 1986;123(2):195–203. 21. The correct answer is B. All of the listed choices are primary determinants of survival from melanoma except maximum diameter of the primary lesion. In addition, the site of distant disease (if present) is also a determinant. Ballo MT, Burmeister BH.  Malignant melanoma. In: Gunderson LL, Tepper JE, editors. Clinical radiation oncology. 4th ed. Philadelphia: Elsevier Saunders; 2016. p. 777–87. 22. The correct answer is C. • Ultraviolet exposure has been linked to melanoma development. –– Higher numbers of melanoma are seen in those ◦◦ With history of sun exposure ◦◦ Who live in high-ambient sunlight areas • Patients with CDKN2A mutation → 70% chance of developing melanoma by age 80. –– Few people with melanoma actually have this mutation.

2  Skin Cancer and Uveal Melanoma

65

• BRAF V600E mutation is present in over half of melanoma cases. • There is an increasing risk of melanoma development in those with an increasing number of nevi. Beral V et al. Cutaneous factors related to the risk of malignant melanoma. Br J Dermatol. 1983;109:165–72. Gellin GA et al. Malignant melanoma. A controlled study of possibly associated factors. Arch Dermatol. 1969;99:43–8. English DR et al. Sunlight and cancer. Cancer Causes Control. 1997;8:271–83. Hussussian CJ et al. Germline p16 mutations in familial melanoma. Nat Genet. 1994;8:15–21. Aitken J et al. CDKN2a variants in a population-based sample of Queensland families with melanoma. J Natl Cancer Inst. 1999;3:446–52. Bishop DT et al. Geographical variation in the penetrance of CDKN2a mutation in melanoma. J Natl Cancer Inst. 2002;94:894–903. Bliss JM et al. Risk of cutaneous melanoma associated with pigmentation characteristics and freckling: systematic overview of 10 case controlled studies. The International Melanoma Analysis Group (IMAGE). Int J Cancer. 1995;62:367–76. 23. The correct answer is D. The ABCDE’s of melanoma are recognized signs for early diagnosis. • A = asymmetry of a lesion. • B = border irregularity. • C = color variation of a lesion. • D = lesion >6 mm, not 5 mm. • E = evolution (or notable changes) in the lesion. Ballo MT, Burmeister BH.  Malignant melanoma. In: Gunderson LL, Tepper JE, editors. Clinical radiation oncology. 4th ed. Philadelphia: Elsevier Saunders; 2016. p. 777–87. 24. The correct answer is A. • Superficial spreading is the most common variant, accounting for 70% of cases. –– More common in women. –– Often arises in a junctional nevus. –– As the malignant cells resemble the cells of Paget’s disease, this is also called pagetoid melanoma. –– Enlargement of the lesion entails invasion of the dermis and subcutaneous tissues by clusters of malignant cells. • Nodular melanoma is the second most common variant (15–25%). –– More common in men –– Often arising on the trunk, head, or neck of middle-aged persons –– Present as raised blue-black lesions. • Lentigo maligna melanoma is less common (10%). –– More common in white women >50 years –– Occurs mostly on the face or neck of white –– Arises from a lentigo maligna (precursor lesion) –– Present as tan, flat lesions that have been present for many years

66

R. F. Thompson et al.

• Acral lentiginous melanoma. –– More common in dark-skinned persons. –– Occurs on the palms, soles, or under the nail beds. Clark WH et al. The developmental biology of primary human malignant melanoma. Semin Oncol. 1975;2:83–103. McCovern VJ, Murad TM. Pathology of melanoma: an overview. In: Balch CM, Milton GW, editors. Cutaneous melanoma: clinical management and treatment results worldwide. Philadelphia: JB Lippincott; 1985. p.  29–53. Clark WH, Mihm MC.  Lentigo maligna and lentigo-­maligna melanoma. Am J Pathol. 1969;55:39–67. Arrington JH et al. Plantar lentiginous melanoma: a distinctive variant of human cutaneous malignant melanoma. Am J Surg Pathol. 1977;1:131–43. Seiji M, Takahashi M. Acral melanoma in Japan. Hum Pathol. 1982;13:607–45. 25. The correct answer is D. Acral lentiginous melanoma is the most likely variant to be found under the nail of the thumbs or great toes. Ballo MT, Burmeister BH.  Malignant melanoma. In: Gunderson LL, Tepper JE, editors. Clinical radiation oncology. 4th ed. Philadelphia: Elsevier Saunders; 2016. p. 777–87. 26. The correct answer is D. Melanoma staging has used two microstaging systems. • Clark level classifies lesions by level of invasion. • Breslow system classifies lesions by thickness: –– More accurate in predicting outcome. –– Part of AJCC staging. –– Ulceration is denoted by suffix “b,” while “a” denotes lack of ulceration. –– Staging ◦◦ T1 refers to lesions ≤1.0 mm ◦◦ T2 lesions are 1.01–2.0 mm thick Gershenwald JE, Scolyer RA, Hess KR et  al. (Eds.). AJCC Cancer Staging Manual (8th edition). Springer International Publishing: American Joint Commission on Cancer; 2017 27. The correct answer is A. • Clark level staging is used only to describe thin T1 tumors. • Clark level II – Invasion but not filling the papillary dermis. • Clark level III  – Filling of the papillary dermis, with compression of the reticular dermis Ballo MT, Burmeister BH.  Malignant melanoma. In: Gunderson LL, Tepper JE, editors. Clinical radiation oncology. 4th ed. Philadelphia: Elsevier Saunders; 2016. p. 777–87. 28. The correct answer is D. Sentinel lymph node biopsy indications: • Melanoma >1.0 mm thick.

2  Skin Cancer and Uveal Melanoma

67

• For lesions 1 mm. –– Wide margins are important for decreasing LR. ◦◦ Lesions 1 mm thick > 2 cm of clear margins –– Adjuvant radiotherapy is not frequently employed as recurrence is usually 4 mm, head or neck site, and ulceration; satellitosis increases this risk to ~15%. Veronesi U, et al. Thin stage I primary cutaneous malignant melanoma. Comparison of excision with margins of 1 or 3 cm [Erratum: N Engl J Med. 1991;325:292] N Engl J Med. 1991;318:1159–62. Thomas JM et al. Excision margins in high-risk malignant melanoma. N Engl J Med. 2004;350:757–66. Heaton KM et al. Surgical margins and prognostic factors in patients with thick (>4 mm) primary melanoma. Ann Surg Oncol. 1998;5:322–28. 30. The correct answer is C. • There is no randomized data that shows a survival benefit for elective nodal dissection at the time of initial surgery. • Sentinel lymph node (SLN) biopsy has largely replaced upfront elective nodal dissection. –– SLN generally is able to predict which patients will benefit from nodal dissection, though this has not been shown to improve survival. –– The randomized Phase III MSLT-II trial: ◦◦ Completion lymph node dissection (LND) following SLNB compared to observation in patient with sentinel node metastasis • No improvement in melanoma-specific overall survival with LND. • Benefit was observed in regional disease control. • A nonrandomized study from the Sydney Melanoma Unit showed a survival benefit from elective nodal dissection in patients with intermediate thickness (0.76–4 mm).

68

R. F. Thompson et al.

• A retrospective review of 157 patients with head and neck melanoma from MD Anderson Cancer Center who underwent elective nodal irradiation instead of dissection: –– Exhibited excellent regional control, 89% at 10 years –– Despite 40 patients harboring occult regional metastases Milton GW et al. Prophylactic lymph node dissection in clinical stage I cutaneous malignant melanoma: results of surgical treatment in 1319 patients. Br J Surg. 1982; 49: 2420–30. Doubrovsky A et al. Sentinel node biopsy provides more accurate staging than elective lymph node dissection in patients with cutaneous melanoma. Ann Surg Oncol. 2004;11:829–36. Ang KK et  al. Postoperative radiotherapy for cutaneous melanoma of the head and neck region. Int J Radiat Oncol Biol Phys. 1994;30:795–98. Faries MB, Thompson JF, Cochran AJ, et al.: Completion dissection or observation for sentinel-node metastasis in melanoma. N Engl J Med. 2017;376:2211–22. 31. The correct answer is A. Of the listed scenarios, a single 1.5 cm lymph node found on elective lymph node dissection is least likely to benefit from adjuvant nodal irradiation. Factors that increase the risk for regional recurrence warranting nodal radiation: • Extracapsular extension of lymph node (strongest predictor) • Size of lymph node >3.0 cm • ≥4 involved lymph nodes • Need for therapeutic dissection • Need for repeat dissection • Location of lymph node in cervical basin In all cases, the potential therapeutic benefit of radiotherapy must be balanced against the potential increase in morbidity, such as extremity edema. Calabro A, et al., Patterns of relapse in 1001 consecutive patients with melanoma nodal metastases. Arch Surg. 1989;124:1051–5. Monsour PD et al. Local control following therapeutic nodal dissection for melanoma. J Surg Oncol. 1993;54:18–22. Miller EJ et al. Loco-regional nodal relapse in melanoma. Surg Oncol. 1992;1:333–40. Bowsher WG et  al. Morbidity, mortality and local recurrence following regional node dissection for melanoma. Br J Surg. 1986;73:906–8. 32. The correct answer is A. Talimogene laherparepvec (also known as Imlygic): • Genetically modified herpes simplex virus type 1 • Designed to replicate within tumors and produce an immunostimulatory protein called granulocyte-macrophage colony-stimulating factor (GM-CSF). • It is known to cause cell lysis, or death, which ruptures tumors, releasing tumor-derived antigens, which along with GM-CSF, may promote an antitumor immune response. –– However, the exact mechanism of systemic action is unknown.

2  Skin Cancer and Uveal Melanoma

69

• It is the first FDA-approved oncolytic viral therapy in the United States. • Approval was based on OPTiM trial –– Phase 3, multicenter, open-label, randomized clinical trial comparing Imlygic to GM-CSF –– 436 patients with advanced melanoma (Stage IIIB, IIIC, or IV) that was not surgically resectable. –– Primary endpoint: Durable response rate (DRR) ◦◦ Percent of patients with complete response (CR) or partial response (PR) maintained continuously for a minimum of 6 months. –– Results ◦◦ DRR 16.3% vs 2.1% among patients treated with Imlygic and GM-CSF, respectively (p 30% rate: muscle spasms, alopecia, dysgeusia (taste disturbance), weight loss, and fatigue. ◦◦ Serious adverse events were reported in 25% of patients. ◦◦ Seven deaths due to adverse events were noted. Cetuximab is a chimeric monoclonal antibody which is utilized as an EGFR inhibitor. Erlotinib is an EGFR tyrosine kinase inhibitor used heavily in lung cancer therapy. Panitumumab is a chimeric monoclonal antibody which is utilized as an EGFR inhibitor, and it differs from cetuximab as it is of the IgG2 type as opposed to the IgG1 subtype. Sekulic A et al. Efficacy and safety of vismodegib in advanced basal-cell carcinoma. N Engl J Med. 2012;366(23):2171–9. Bean J et  al. MET amplification occurs with or without T790M mutations in EGFR mutant lung tumors with acquired resistance to gefitinib or erlotinib. Proc Natl Acad Sci U S A. 2007;104(52):20932–7. Yan L et al. Pharmacogenetics and p­ harmacogenomics in oncology therapeutic antibody development. Biotechniques. 2005;39(4):565–8.

74

R. F. Thompson et al.

44. The correct answer is A. Please refer to Question 43 for further information. Basal cell carcinoma exhibits dysregulation of the sonic hedgehog pathway with activating mutations in Smoothened (SMO). Sonic hedgehog (SHH) is a ligand which binds another protein Patched-1 (PTCH), whose function is to inhibit smoothened activation in the absence of SHH. • Normal –– SHH absent → PTCH inhibits SMO → SMO cannot carry out downstream pathways –– SHH present → PTCH does not inhibit → SMO can carry out downstream pathways • Abnormal –– Does not matter whether SHH is present or not –– PTCH cannot do anything to SMO –– Mutated SMO carries out downstream pathways • Vismodegib inhibits the mutant SMO Sekulic A et al. Efficacy and safety of vismodegib in advanced basal-cell carcinoma. N Engl J Med. 2012;366(23):2171–9. 45. The correct answer is D. • SCC and BCC account for 95% of skin cancer in organ transplant recipients. • SCC is the most common type of skin cancer in the immunocompromised population. –– Reverse of what is observed in the immunocompetent population • SCC incidence is increased by 65-fold relative to the normal population. • The risk of SCC development in OTRs increases steadily with time posttransplant. –– In the US and western Europe ◦◦ 10 years posttransplant: 10%–27% ◦◦ 20 years posttransplant: 40%–60% • These tumors are more aggressive with increased morbidity and mortality, and some patients develop a catastrophic number of tumors. O’Reilly Zwald F et  al. Skin cancer in solid organ transplant recipients: advances in therapy and management: part I. Epidemiology of skin cancer in solid organ transplant recipients. J Am Acad Dermatol. 2011;65(2):253–61. 46. The correct answer is B. A thorough understanding of head and neck anatomy is vital in the setting of perineural invasion. For cutaneous squamous cell carcinoma of the malar cheek, focus is placed on coverage on the maxillary division of the trigeminal nerve (V2). Coverage of the nerve pathway from distal to proximal is the following: • Infraorbital nerve which exits the infraorbital foramen below the orbit –– Travels along the infraorbital foramen until it leaves through the inferior orbital fissure (IOF) • Maxillary nerve –– Designation when infraorbital nerve crosses the IOF

2  Skin Cancer and Uveal Melanoma

75

–– Maxillary nerve crosses the pterygopalatine fossa (PPF) moving superiorly –– Stops at foramen rotundum Ko HC et  al. A contouring guide for head and neck cancers with perineural invasion. Pract Radiat Oncol. 2014;4(6):e247–58. 47. The correct answer is C. • The lower lip innervation proximal to distal: –– Mental nerve – exterior to the mandible from the mental foramen –– Inferior alveolar nerve  – when the mental nerve travels proximally through the mental foramen of the mandible ◦◦ Travels inside the mandible and exits proximally through mandibular foramen –– Mandibular nerve – when IAN exits mandibular foramen ◦◦ Follows the posterior border of the masticator space at the division of the lateral and medial pterygoid muscles ◦◦ The mandibular nerve enters the skull at the foramen ovale Ko HC et  al. A contouring guide for head and neck cancers with perineural invasion. Pract Radiat Oncol. 2014;4(6):e247–58. 48. The correct answer is D. Per “Definitive and Postoperative Radiation Therapy for Basal and Squamous Cell Cancers of the Skin: An ASTRO Clinical Practice Guideline,” a consensus deemed the following indications for definitive radiation for basal cell carcinoma and squamous cell carcinoma: • Patients who cannot undergo or decline surgical resection. • Areas where surgery can compromise function or cosmesis. –– Ears –– Nasal ala –– Nose –– Lips including oral commissure –– Eyelids including medial and lateral canthi • Conditionally NOT recommended for patients with genetic disease predisposing to heightened radiosensitivity. –– Gorlin syndrome ◦◦ The patient in choice D has odontogenic keratocysts in the setting of Gorlin syndrome. Although his lesion is at the ala, feasibility and prescription of non-radiation options should be exhausted before pursuing further management with radiation –– Ataxia-telangiectasia –– Xeroderma pigmentosum –– Li-Fraumeni syndrome Likhacheva A et al. Definitive and Postoperative Radiation Therapy for Basal and Squamous Cell Cancers of the Skin: Executive Summary of an American Society for Radiation Oncology Clinical Practice Guideline. Pract Radiat Oncol. 2020;10(1):8–20.

76

R. F. Thompson et al.

49. The correct answer is C. Per “Definitive and Postoperative Radiation Therapy for Basal and Squamous Cell Cancers of the Skin: An ASTRO Clinical Practice Guideline,” a consensus deemed the following strong indications for postoperative radiation for basal cell carcinoma and squamous cell carcinoma: • Gross perineural spread that is clinically or radiologically apparent • Close or positive margins that cannot be correct with further surgery • Recurrence after a prior margin resection • T3 or T4 tumors • Desmoplastic or infiltrative tumors in the setting of chronic immunosuppression • Among the answer choices list, C is incorrect as this would be classified as a T2 lesion and would not have a strong indication for postoperative radiation therapy. Likhacheva A et al. Definitive and Postoperative Radiation Therapy for Basal and Squamous Cell Cancers of the Skin: Executive Summary of an American Society for Radiation Oncology Clinical Practice Guideline. Pract Radiat Oncol. 2020;10(1):8–20. 50. The correct answer is C. Brantsch et al. • Prospective study of potential risk factors for metastasis or local recurrence of SCC • 615 of 653 assessable patients • Treatment –– Period: Between Jan 1, 1990, and Dec 31, 2001. –– All patients underwent surgery for cutaneous SCC with complete histological examination of the three-dimensional excision margins (3D-­histology) in one center. –– Primary endpoints ◦◦ Time to metastasis ◦◦ Time to local recurrence –– Desmoplasia definition ◦◦ Fine branches of tumor cells at the periphery and surrounding stromal reaction ◦◦ Perineural or perivascular invasion were always associated with desmoplasia ◦◦ All SCC in which at least 1/3rd of the specimen met these criteria were classified as desmoplastic SCC –– Results ◦◦ Median follow-up – 43 months ◦◦ Metastases – 26 (4%) patients ◦◦ Local recurrence – 20 (3%) patients

2  Skin Cancer and Uveal Melanoma

77

• 13/20 (60%) occurred within the 1st year • Risk with and without desmoplasia, respectively: 24% vs 1% • Risk with tumor thickness  ≤  6.0 and 6.0  mm, respectively: 12% vs 3% Brantsch KD et al. Analysis of risk factors determining prognosis of cutaneous squamous-cell carcinoma: a prospective study. Lancet Oncol. 2008;9(8):713–20. 51. The correct answer is D. • Radiation to the lacrimal gland can lead to permanent reduction of goblet cells and loss of serous acinar cells. This leads to decreased tear production and altered consistency of tears: • Complications include: –– Corneal vascularization and opacification –– Keratoconjunctivitis sicca • Dose >57  Gy can lead to the above symptoms within 10  months in all patients • Sreeraman et al. –– Clinical-dosimetric analysis of lacrimal gland dysfunction in patient treated with IMRT for sinonasal tumors –– Results ◦◦ Among measurements of maximum dose, mean dose, V10, V20, and V30, a V30 5 cm ◦◦ Invading surrounding cartilage, skeletal muscle, or bone ◦◦ In-transit metastases • Results: –– Median follow-up: 60 months –– Median RT dose: 60 Gy –– 2 and 5 year freedom from locoregional relapse (FFFLR): ◦◦ RT alone: 88% and 83%, respectively ◦◦ ChemoRT: 89% and 87% ◦◦ No statistically significant difference (p = 0.58) • 5 year OS (~75–80%) and DFS (~70%) were not significantly different • No difference in toxicity Porceddu SV et  al. Postoperative Concurrent Chemoradiotherapy Versus Postoperative Radiotherapy in High-Risk Cutaneous Squamous Cell Carcinoma of the Head and Neck: The Randomized Phase III TROG 05.01 Trial. J Clin Oncol. 2018;36(13):1275–83.

2  Skin Cancer and Uveal Melanoma

81

57. The correct answer is C. • The original large prospective trial, the COMS trial, used I-125. –– Consequently, most US centers use I-125. • However, other sources are also used. –– For example, Ru-106 may be superior for tumors 11 mm maximal basal tumor diameter • >60y/o and ≤11 mm maximal basal tumor diameter • >60y/o and >11 mm maximal basal tumor diameter Collaborative Ocular Melanoma Study (COMS) Group. The COMS Randomized Trial of Iodine 125 Brachytherapy for Choroidal Melanoma. Twelve-Year Mortality Rates and Prognostic Factors: COMS Report No. 28. Arch. Ophthalmol. 2006 59. The correct answer is D. Verma and Mehta systematically reviewed 14 original investigations published from 2000–2015. Dose ranged from 50 to 70.2CGE in 4–5 fractions. Verma V et al. Clinical Outcomes of Proton Radiotherapy for Uveal Melanoma. Oncol (R Coll Radiol). 2016;28(8):e17–27.

82

R. F. Thompson et al.

60. The correct answer is B. Please refer to Question 58 for further details. COMS randomized multicenter trial of I-125 plaque brachytherapy vs enucleation • 12-year follow-up outcomes (brachytherapy vs enucleation, respectively) –– All-cause mortality ◦◦ 12-year all-cause mortality: 43% vs 41% in the enucleation arm –– Death with metastatic melanoma ◦◦ 5 year: 10% vs 11% ◦◦ 10 year: 18% vs 17% ◦◦ 12 year: 21% vs 17% Collaborative Ocular Melanoma Study (COMS) Group. The COMS Randomized Trial of Iodine 125 Brachytherapy for Choroidal Melanoma. Twelve-Year Mortality Rates and Prognostic Factors: COMS Report No. 28. Arch. Ophthalmol. 2006 61. The correct answer is B. • Side effects include optic neuropathy, radiation retinopathy, dry eye, conjunctivitis, cataract, iris neovascularization, glaucoma, maculopathy, scleral necrosis, retinal detachment, hemorrhage, and decreased vision in the ipsilateral eye Groenewald C et al. Effects of radiotherapy on uveal melanomas and adjacent tissues. Eye (Lond). 2013;27(2):163–71.

Chapter 3

Head and Neck Cancer Shushan Rana, John M. Holland, Kristina H. Young, and Celine Bicquart Ord

Questions 1. The incidence of head and neck cancer in the United States is approximately _____________ cases per year. A. 100,000 B. 50,000 C. 30,000 D. 15,000

S. Rana Radiation Oncologist, PeaceHealth Southwest Medical Center, Vancouver, WA, USA e-mail: [email protected] J. M. Holland (*) Department of Radiation Medicine, Oregon Health Science University, Portland, OR, USA e-mail: [email protected] K. H. Young The Oregon Clinic – Radiation Oncology, Portland, OR, USA e-mail: [email protected] C. B. Ord Carson Tahoe Radiation Oncology Associates, Carson City, NV, USA © Springer Nature Switzerland AG 2021 R. A. Chandra et al. (eds.), Radiation Oncology Study Guide, https://doi.org/10.1007/978-3-030-53687-9_3

83

84

S. Rana et al.

2. Which of the following is associated with an improved outcome in oropharyngeal cancer? A. p16 mutation B. Lack of EBV viral load C. p53 mutation D. HPV infection 3. How does the human papillomavirus promote carcinogenesis? A. E7 protein binds p53; E6 protein binds Rb. B. E6 protein binds p53; E7 protein binds Rb. C. E6 protein promotes p16 expression. D. None of the above. 4. All of the following are subsites of the larynx except: A. True vocal cord B. Pyriform sinus C. Aryepiglottic fold D. Suprahyoid epiglottis 5. The most commonly involved subsite of the oropharynx is: A. Soft palate B. Tonsil C. Base of the tongue D. Pharyngeal wall 6. Which of the following nerves is not associated with otalgia related to a mass in the oral tongue, base of the tongue, larynx, or hypopharynx? A. Auriculotemporal branch of CN V B. Jacobson nerve of CN IX C. Posterior auricular nerve of CN VII D. Arnold nerve of CN X 7. What is the approximate risk of lymph node metastases for a T1a true vocal cord cancer? A. 2% B. 7% C. 10% D. 15% 8. Which of the following lymph node levels contains the Delphian node? A. Level III B. Level IV C. Level V D. Level VI

3  Head and Neck Cancer

85

9. What is the primary lymph node drainage for the supraglottic larynx? A. Level II B. Level V C. Level VI D. Level I 10. Of the listed tumor sites, which is least likely to demonstrate a bilateral nodal involvement? A. Soft palate B. Base of the tongue C. Pharyngeal wall D. Tonsil 11. Which of the following skull base structures is matched properly with the CN that traverses it? A. Cavernous sinus: CN II B. Foramen ovale: CN V2 C. Jugular foramen: CN IX D. Foramen lacerum: CN V3 12. Which of the following is not a subsite of the hypopharynx? A. Pyriform sinus B. Postcricoid area C. Posterior pharyngeal wall D. Palatine tonsil 13. The inferior border of the hypopharynx is: A. C2 B. Hyoid C. Cricoid D. Sternal notch 14. What structure anatomically divides the superficial and deep parotid? A. CN V2 B. Zygomatic branch of CN VII C. Buccal branch of CN VII D. Mandibular branch of CN VII 15. A 68-year-old lady presents with a 1.5  cm p16-positive squamous cell carcinoma of the R tonsil resected via transoral robotic surgery. An ipsilateral R selective neck dissection of II–IV demonstrated a 2.5 cm lymph node with ECE 3 cm B. ≥2 involved lymph nodes C. Perineural invasion D. Extracapsular extension 17. In the postoperative setting, what is the most appropriate dose in a nodal region associated with extracapsular spread? A. 54 Gy B. 57.6 Gy C. 63 Gy D. 68.4 Gy 18. GORTEC 94-01 demonstrated a benefit to concurrent chemoradiation over radiation alone in advanced oropharynx cancer in terms of: A. Local control but not overall survival benefit. B. Local control and overall survival benefit. C. Local control and distant-metastasis-free benefit. D. There was no benefit. 19. Based on the combined analysis of EORTC 22931 and RTOG 9501, which two pathologic findings are associated with a chemotherapy benefit? A. Perineural invasion, positive margins B. Angiolymphatic invasion, positive margins C. ≥2 involved lymph nodes, perineural invasion D. Extracapsular extension, positive margins 20. Which of the following is a correct pairing for anatomic boundaries of lymph node stations in the neck? A. Level II – caudal edge of medial pterygoid to platysma B. Level III – bottom edge of cricoid to clavicle C. Level V – skull base to clavicle D. Level VI – bottom edge of cricoid to clavicle 21. What were the reported 3-year overall survivals with and without concurrent cisplatin, respectively, in patients with unresectable head and neck squamous cell carcinoma, based on the intergroup study reported by Adelstein et al.? A. 61% versus 42% B. 57% versus 44% C. 37% versus 23% D. 27% versus 14%

3  Head and Neck Cancer

87

22. The addition of concurrent cetuximab to radiation offers which of the following benefits, based on the Bonner NEJM 2006 data: A. Local control but not overall survival benefit. B. Local control and progression-free survival but not overall survival benefit. C. Local control, progression-free, and overall survival benefit. D. There was no benefit. 23. What is the approximate 3-year OS for patients receiving the Posner induction regimen of TPF (docetaxel, cisplatin, 5-FU) followed by concurrent carboplatin and radiation for unresectable H&N cancer? A. 70% B. 60% C. 50% D. 40% 24. Which of the following is true regarding the DeCIDE trial which compared induction chemotherapy followed by concurrent chemoradiation versus chemoradiation alone? A. The chemotherapy paired with concurrent radiation was carboplatin. B. Induction chemotherapy consisted of three cycles of chemotherapy. C. Induction therapy followed by chemoradiation improves OS, recurrence-­ free survival, and disease-free survival. D. Adverse events are more common with induction chemotherapy. 25. Which of the following is true regarding PARADIGM trial on the role of induction chemotherapy vs concurrent chemoradiation? A. Induction chemotherapy improved 3-year OS and PFS. B. Induction chemotherapy improved 3-year PFS but not OS. C. Febrile neutropenia incidence increased with induction chemotherapy. D. The chemotherapy in the concurrent chemoradiation-alone arm was carboplatin. 26. Based on the MACH-NC meta-analysis, what are the hazard ratios (HR) for death compared to RT alone with concurrent chemoradiation and induction chemotherapy and then radiation, respectively? A. Concurrent HR 1.0; induction HR 1.0 B. Concurrent HR 0.8; induction HR 0.6 C. Concurrent HR 0.8; induction HR 1.0 D. Concurrent HR 0.6; induction HR 0.8 27. Hyperfractionated radiation for H&N cancers has demonstrated a benefit in: A. Local control B. Overall survival C. Disease-free survival D. A and B only

88

S. Rana et al.

28. To what dose should you limit the mandible to minimize the risk of osteoradionecrosis? A. 50 Gy B. 60 Gy C. 70 Gy D. 80 Gy 29. Which of the following is effective for reducing toxicity associated with H&N radiation? A. Viscous lidocaine B. Limiting the parotid mean dose to 2 cm and ≤ 3 cm Carter BW, et al. Revisions to the TNM staging of lung cancer: rationale, significance, and clinical application. Radiographics. 2018;38(2):374–391. 30. The correct answer is C. • Involvement of the main bronchi: –– In TNM-7 ◦◦ Involvement of main bronchus 2 cm or more from the carina = T2. ◦◦ More proximal involvement = T3. –– Analysis of the new data demonstrated for AJCC 8th edition ◦◦ Multivariate analysis revealed that involvement of a main bronchus, regardless of the distance from the carina, does not increase risk after adjusting for tumor size. ◦◦ Involvement main bronchus, regardless of the distance from the carina = T2 AJCC 8th edition. • Atelectasis or pneumonitis (of a lung lobe) –– In TNM-7 ◦◦ Partial atelectasis or pneumonitis = T2. ◦◦ Total lung atelectasis or pneumonitis = T3. –– Analysis of the new data demonstrated for AJCC 8th edition ◦◦ Partial atelectasis or pneumonitis aligns with other T2 descriptors with regard to 5-year survival. ◦◦ Patients with complete atelectasis or pneumonitis demonstrated better survival than those with other T3 descriptors. ◦◦ In TNM-8th edition, partial and complete forms of lung atelectasis and pneumonitis are grouped together as T2 lesions. • Diaphragmatic invasion –– In TNM-7 ◦◦ Invasion of the diaphragm by lung cancer = T3. –– Analysis of the new data for AJCC 8th edition ◦◦ Worse 5-year survival of patients with this feature than that of patients with other T3 tumors. ◦◦ Similar to the 5-year survival of patients with T4 lesions. ◦◦ Therefore, diaphragmatic invasion is reclassified as T4 disease in TNM-8.

4  Thoracic Cancer

191

• Mediastinal pleura invasion –– In TMN-7 ◦◦ Involvement of the mediastinal pleura = T3. –– Analysis of the new data for AJCC 8th edition ◦◦ Patients with mediastinal pleura invasion have better prognosis than patients with other T3 lesions, although this characteristic was represented by a relatively small number of cases. ◦◦ In clinical practice, mediastinal pleura invasion can be considered if the lesion directly contacts the mediastinum. ◦◦ However, when other findings suggestive of invasion are present, the tumor has typically already extended beyond the mediastinal pleura and invaded the mediastinum, a finding that remains classified as T4 disease. ◦◦ At pathologic staging, the discovery of isolated mediastinal pleura invasion in the absence of invasion of additional mediastinal structures is rare. ◦◦ Key point: mediastinal pleural invasion has been eliminated from the T classification in TNM-8. Carter BW, et al. Revisions to the TNM staging of lung cancer: rationale, significance, and clinical application. Radiographics. 2018;38(2):374–391. 31. The correct answer is B. Nodules in different lobes of the ipsilateral lung are classified as T4 in the AJCC 8th edition. Carter BW, et al. Revisions to the TNM staging of lung cancer: rationale, significance, and clinical application. Radiographics. 2018;38(2):374–391. 32. The correct answer is A. In the AJCC 8th edition, supraclavicular lymph node metastases, whether they are ipsilateral or contralateral, are classified as N3. Carter BW, et al. Revisions to the TNM staging of lung cancer: rationale, significance, and clinical application. Radiographics. 2018;38(2):374–391. 33. The correct answer is C. In the AJCC 8th edition • T4 tumors are classified as those with: –– Size >7 cm –– Direct invasion into the: ◦◦ Mediastinum ◦◦ Chest wall ◦◦ Diaphragm ◦◦ Heart ◦◦ Great vessels ◦◦ Trachea ◦◦ Recurrent laryngeal nerve ◦◦ Esophagus ◦◦ Vertebral body ◦◦ Carina

192

S. Rana et al.

• Separate tumor nodule or nodules in a different lobe of the same lung • N1 nodal stage: ipsilateral hilar lymph node involvement • M1a: –– Separate tumor nodules or nodules in the contralateral lung –– Malignant pleural effusions or pleural thickening or nodules or masses –– Malignant pericardial effusion or pericardial thickening or nodules or masses Carter BW, et al. Revisions to the TNM staging of lung cancer: rationale, significance, and clinical application. Radiographics. 2018; 38(2):374–391. 34. The correct answer is B. Adenocarcinoma • Most commonly (75%) presents in a peripheral location. • Can arise out of old tuberculosis scars. • Slowest doubling time of non-small-cell lung cancer subtypes. • Stage-for-stage has worse outcomes than squamous cell carcinoma. Wagner H. Non-small cell lung cancer. In: Gunderson LL, Tepper JE, editors. Clinical radiation oncology. 3rd ed. Philadelphia: Elsevier Saunders; 2012. p.  805–38. Arai T, et  al. Tumor doubling time and prognosis in lung cancer patients: evaluation from chest films and clinical follow-up study. Jpn J Clin Oncol. 1994;24(4):199–204. 35. The correct answer is A. Of the listed subtypes of lung cancer, bronchioalveolar carcinoma is least related to smoking. Ebbert JO, et al. Clinical features of bronchioalveolar carcinoma with new histologic and staging definitions. J Thoracic Oncol. 2010;5(8):1213–20. Wagner H Jr, Langer JC.  Non-small cell lung cancer. In: Gunderson LL, Tepper JE, editors. Clinical radiation oncology. 4th ed. Philadelphia: Elsevier Saunders; 2016. p. 809–43. 36. The correct answer is D. • More than 5000 patients with inoperable bronchogenic carcinoma of the lung were entered in Veterans Administration Lung Group Protocols 9–15. • Three most important prognostic factors affecting survival: –– Karnofsky performance status –– Extent of disease –– Weight loss in the previous 6 months Stanley KE, et  al. Prognostic factors for survival in patients with inoperable lung cancer. J Natl Cancer Inst. 1980;65(1):25–32. 37. The correct answer is B. Lung Cancer Study Group trial • T1N0M0 lung cancer • Randomization –– Lobectomy –– Limited resection

4  Thoracic Cancer

193

• Results –– Significantly improved local control in those that underwent lobectomy –– Cancer mortality was increased in those that underwent limited resection ◦◦ 62% versus 55% ◦◦ Wedge resection associated with three-fold increase in local recurrence ◦◦ Segmental resection associated with 2.4-fold increase in  local recurrence –– Of those that underwent limited resection, none required postoperative ventilation >24 h. • Conclusion: Given the increased local recurrence and cancer mortality associated with limited resection, lobectomy should remain the standard surgical treatment. Ginsberg R, Rubenstein L. Randomized trial of lobectomy versus limited resection for T1N0 non-small cell lung cancer. Lung Cancer Study Group. Ann Thorac Surg. 1995; 60:615–23. 38. The correct answer is A. • There is data showing that SBRT is an excellent therapy for medically inoperable early-stage lung tumors. • Surgical resection remains the gold standard for operable NSCLC. Ginsberg R, Rubenstein L. Randomized trial of lobectomy versus limited resection for T1N0 non-small cell lung cancer. Lung Cancer Study Group. Ann Thorac Surg. 1995; 60:615–23. 39. The correct answer is D. Mountain data • 5319 combined consecutive patients with lung cancer. –– 4351 treated at M.D. Anderson Cancer Center –– 968 from the Reference Classification for Anatomic and Pathologic Classification of Lung Cancer database • Include clinical, surgical, pathologic, and follow-up data. • Results –– 5-year OS of cT1N0 lung cancer is 61%. –– 5-year OS of pT1N0 lung cancer is 67%. Mountain CF. Revisions in the international lung cancer system for staging lung cancer. Chest. 1997;111(6):1710–7. 40. The correct answer is C. Please refer to question 39. Mountain CF. Revisions in the international lung cancer system for staging lung cancer. Chest. 1997;111(6):1710–7. 41. The correct answer is C. RTOG 0236 • Phase II study –– SBRT in 55 patients with early-stage, medically inoperable NSCLC patients. –– Both T1 and T2 tumors were included.

194

S. Rana et al.

• Treatment –– Prescription dose 20 Gy × 3 fractions without heterogeneity corrections. ◦◦ Dose delivered in actuality: ~18 Gy × 3. • Results –– 3-year primary tumor control rate: 97.6% –– 3-year distant failure: 22% –– 3-year OS: 55.8% Timmerman R et  al. Stereotactic body radiation therapy for inoperable early stage lung cancer. JAMA. 2010;303(11):1070–6. 42. The correct answer is B. Please refer to question 41 for further details RTOG 0236 long-term results • Median follow-up of 4.0 years (7.2 years for surviving patients) • 5-year primary tumor failure rate: 7% • 5-year local (primary tumor and involved lobe) failure rate: 20% • 5-year local-regional failure rate: 38% • 5-year disseminated failure rate: 31% • 5-year DFS: 26% • 5-year OS: 40% • mOS: 4 years Timmerman R, et al. Long-term results of RTOG 0236: a phase II trial of stereotactic body radiation therapy (SBRT) in the treatment of patients with medically inoperable stage I non-small cell lung cancer. Int J Radiat Oncol Biol Phys. 2014;90:S30. 43. The correct answer is C. Onishi et al. on final BED of SBRT in lung cancer • 257 patients with operable stage I NSCLC • Hypofractionated radiation –– Doses ranging 18–75 Gy at the isocenter in 1–22 fractions • Results –– BED ≥100 Gy compared to 15 centroblasts/HPF Centrocytes present Solid sheets of centroblasts

Treatment ISRT ± anti-CD20 ± chemo ISRT ± anti-CD20 ± chemo ISRT ± anti-CD20 ± chemo As G1–2, or G3B As DLBCL

NCCN Guidelines for B-Cell Lymphomas, Version 1.2020

538

B. A. Dyer et al.

89. The correct answer is C. • The FL-ILROG series is a retrospective cohort of 512 early-stage patients treated between 2000 and 2017. Patients were staged with PET/CT; ~80% were stage I. Patients were treated with a variety of radiation techniques and doses ≥24 Gy. The 5-year local control was 91.9%, and in-field or marginal relapses were 2.2%. • Treatment outcomes after RT in patients staged with PET/CT are better than earlier series  – particularly for stage I patients  – suggesting that cure is potential for localized FL. Brady JL, et  al. Definitive radiotherapy for localized follicular lymphoma staged by 18F-FDG PET-CT: a collaborative study by ILROG.  Blood. 2019;133(3):237–45. 90. The correct answer is C. • The FORT trial was a randomized phase III non-inferiority study (≤10% difference in  local progression at 2  years) for patients with indolent lymphoma comparing 4  Gy/2 fx vs 24  Gy/12 fx, and nearly 60% of patients were treated with palliative intent. • Complete response was noted in 68% of patients treated with 24  Gy and 49% of patients treated with 4 Gy. • Of patients treated with curative intent, complete response was seen in 75% of patients treated with 24 Gy and 58% of patients treated with 4 Gy. • PFS is with 4 Gy is inferior to 24 Gy for indolent B-cell non-Hodgkin lymphoma, with no difference in survival outcomes; therefore, 24 Gy is the standard of care, but 4 Gy is useful for palliative treatment. Hoskin PJ, et  al. 4  Gy versus 24  Gy radiotherapy for patients with indolent lymphoma (FORT): a randomised phase 3 non-inferiority trial. Lancet Oncol 2014; 15: 457–63. 91. The correct answer is A. • The expected objective response rate for patients with indolent lymphoma – even with bulky disease – is >90%; complete response is expected in ~60%, and partial response is expected in ~30%. Haas RL, et  al. High response rates and lasting remissions after low-dose involved field radiotherapy in indolent lymphomas. J Clin Oncol. 2003;21(13):2474–80. 92. The correct answer is B. • Gastric lymphoma often presents with multifocal disease requiring whole-­ stomach irradiation. • The patient should be simulated in the supine position with arms up and a custom mold for immobilization. • The patient should fast 2–4 hours prior to simulation and treatment. • While not absolutely necessary, oral contrast can help define the gastric wall from the adjacent colon, pancreas, and small bowel. Small-volume contrast (25–30 cc or ~1 oz) should be swallowed immediately prior to CT imaging.

12  Heme Malignancies

539

• IV contrast should be considered if lymph nodes are involved. • 3D, IMRT, and VMAT are acceptable treatment techniques. • 4D-CT can be useful to account for respiratory motion and margin reduction; surface imaging and/or respiratory motion management may also be useful; however, abdominal compression should be avoided due to variation in day-to-day setup. 93. The correct answer is D. • The GTV should be delineated if possible, based on tumor visible on imaging or the description of the EGD note. Oral contrast (~1 oz) can help with the visualization of tumor. • If using a 4D-CT, the ITV should account for both motion of the stomach and any involved nodes. • The CTV should cover the GTV/ITV with 1–2 cm margin; if there is multifocal disease, the entire stomach, nodes (if any), and several centimeters of the gastroduodenal junction (common site of H. pylori infection) should be covered. • The PTV margin of 1–2 cm is reasonable depending on patient factors, treatment technique, patient setup and motion management, and daily imaging QA. • 3D treatment plans use a three-field box, including opposed lateral and AP beams, and the kidneys should be blocked. • Patients should be placed on a PPI and antiemetic during radiation. • Daily CBCT should be used for IMRT/VMAT plans or daily kv/kv for 3D plans with consideration of weekly CBCT.

Chapter 13

Soft Tissue Sarcomas and Bone Tumors Christina Binder, Celine Bicquart Ord, and Arthur Hung

Questions 1. All of the following are true regarding soft tissue sarcoma, except: A. They are derived from mesenchymal tissue. B. Median age of onset is 50–55 years. C. Five-year survival for all sites and stages combined is 50–60%. D. Local progression is the mode of death for extremity and torso sarcomas. 2. What syndrome is associated with the development of intra-abdominal desmoid tumors? A. Gardner’s syndrome B. Cowden’s syndrome C. Ataxia telangiectasia D. Nelson’s syndrome

C. Binder (*) Spears Cancer Center, Grants Pass, OR, USA Department of Radiation Medicine, Oregon Health Science University, Portland, OR, USA e-mail: [email protected] C. B. Ord Carson Tahoe Radiation Oncology Associates, Carson City, NV, USA A. Hung Department of Radiation Medicine, Oregon Health Science University, Portland, OR, USA e-mail: [email protected] © Springer Nature Switzerland AG 2021 R. A. Chandra et al. (eds.), Radiation Oncology Study Guide, https://doi.org/10.1007/978-3-030-53687-9_13

541

542

C. Binder et al.

3. What translocation is characteristic of synovial cell sarcoma? A. t(9;22) B. t(12;16) C. t(2;13) D. t(X;18) 4. What translocation is characteristic of Ewing’s sarcoma? A. t(11;22) B. t(12;16) C. t(9;22) D. t(X;18) 5. All are true regarding Li-Fraumeni syndrome, except: A. It results from a germ-line mutation of p53. B. There is a 50% risk of malignancy by age 70. C. There is a risk of developing breast cancer. D. There is a risk of developing osteosarcoma. 6. Which is the most common malignancy associated with hereditary retinoblastoma? A. Osteosarcoma B. Pineoblastoma C. Hepatocellular carcinoma D. Colorectal cancer 7. Which is the most common site of soft tissue sarcoma? A. Trunk B. Upper extremity C. Head and neck D. Lower extremity 8. All of the following are true regarding cutaneous angiosarcoma, except: A. Staging is based on the TNM system and also includes grade. B. Fifty percent of cases occur in the head and neck. C. Local recurrence is frequent but distant metastases are not. D. Regional lymph node metastases are more frequent than in other soft tissue sarcomas. 9. All of the following are true regarding dermatofibrosarcoma protuberans, except: A. It exhibits slow, indolent growth. B. It stains for CD 34+. C. It rarely recurs after simple local excision. D. Those with the t(17;22) translocation respond to imatinib. 10. Regarding the NCI soft tissue sarcoma trial of amputation versus limb-sparing surgery + radiation, which of the following is not true? A. Local control in the amputated arm was 100%.

13  Soft Tissue Sarcomas and Bone Tumors

543

B. Total radiation dose was 45–50 Gy. C. All patients received postoperative chemotherapy. D. 15% of limb-sparing patients had positive surgical margins. 11. Regarding the NCI soft tissue sarcoma trial of amputation versus limb-sparing surgery + radiation, which of the following is not true? A. There was no difference in local control between arms. B. Most recurrences in the limb-sparing arm were isolated local failures. C. There was no difference in disease-free survival between the arms. D. There was no difference in overall survival between the arms. 12. What is the local recurrence rate with a planned positive margin after resection? A. 4% B. 8% C. 12% D. 16% 13. What is the local recurrence rate with an unplanned resection elsewhere with a positive margin on re-excision? A. 20% B. 30% C. 40% D. 50% 14. What is the expected local control rate of a sarcoma after surgery and adjuvant radiation? A. 75% B. 80% C. 85% D. 90% 15. In the NCI trial of limb-sparing surgery +/− adjuvant therapy for sarcomas of the extremities, all of the following are true, except: A. No chemotherapy was used in any patients. B. Local control for those receiving radiation was improved. C. There was no difference in local control in low- versus high-grade tumors. D. Those with low-grade tumors were randomized to adjuvant radiation versus observation. 16. In the Memorial Sloan Kettering Cancer Center trial of limb-sparing surgery +/− adjuvant brachytherapy, which of the following is not true? A. Complications were reduced with loading of catheters on post-op day 5 as opposed to immediately postoperatively. B. Total brachytherapy dose was 45 Gy. C. Positive surgical margins predicted for local recurrence. D. A local control benefit for brachytherapy was found only for high-­ grade tumors.

544

C. Binder et al.

17. In the NCI Canada trial of pre- versus postoperative EBRT, all of the following are true, except: A. There was no local control difference between the arms. B. Positive surgical margins predicted for local recurrence. C. Size and grade of the primary tumor did not predict for recurrence-free survival. D. Positive surgical margins were treated with a 16–20 Gy boost. 18. All of the following situations preclude brachytherapy as the sole adjuvant therapeutic modality for soft tissue sarcoma, except: A. The CTV cannot be adequately encompassed in implant geometry. B. Proximity of treatment volume near a critical structure precludes delivery of a meaningful dose. C. Resection margins are positive. D. Skin ulceration is present. 19. Which of the following radiation dose recommendations is incorrect for soft tissue sarcoma of the extremity/trunk/head and neck? A. Preoperative external beam radiation – 50 Gy B. Postoperative external beam radiation for negative margins – 50–60 Gy C. Postoperative HDR brachytherapy alone – 45 Gy D. Boost dose for microscopic positive margins – 16–18 Gy 20. All of the following are true regarding radiation target volumes for soft tissue sarcoma, except: A. Tumor cells are generally found at a maximum of 4 cm from the primary tumor. B. The deep margin still needs to include deep muscle for a superficial tumor. C. Longitudinal margins are longer than radial margins. D. Soft tissue sarcoma generally respects anatomical fascial planes. 21. What translocation is associated with alveolar rhabdomyosarcoma? A. t(2;13) B. t(X;17) C. t(12;22) D. t(X;18) 22. All of the following are true regarding the sequential chemotherapy and radiation neoadjuvant treatment approach described by DeLaney et al., except: A. Five-year overall survival was significantly improved versus historical controls. B. The improvement in overall survival was due to improved local control. C. Radiotherapy consisted of two split courses of 22 Gy. D. Positive margins were treated with a radiotherapy boost. 23. All of the following are true regarding isolated limb perfusion for soft tissue sarcomas, except: A. Chemotherapy consists of melphalan and TNF-α. B. Overall response rates are as high as 95%.

13  Soft Tissue Sarcomas and Bone Tumors

545

C. Limb-preservation rates are in excess of 80%. D. Most appropriate in patients of resection with probable sacrifice of important function. 24. Which of the following was not a finding from the landmark meta-analysis of adjuvant chemotherapy in soft tissue sarcoma by the Sarcoma Meta-analysis Collaboration (Lancet 1997)? A. Distant relapse-free interval was improved with chemotherapy. B. Overall recurrence-free survival was improved with chemotherapy. C. Overall survival was improved with chemotherapy. D. Local recurrence-free interval was improved with chemotherapy. 25. All of the following are true regarding retroperitoneal sarcomas, except: A. These tumors typically present larger in size due to late presentation of symptoms. B. Adjuvant radiation can be easily delivered with the use of IMRT. C. Preoperative radiation does not increase wound complications. D. Intraoperative radiotherapy may improve local control and overall survival. 26. Which of the following is false regarding primary sarcomas of the stomach? A. Gastrointestinal stromal tumors frequently recur with lung metastases. B. Gastrointestinal stromal tumors respond well to imatinib. C. Half of gastrointestinal stromal tumor recurrences are in the liver. D. Complete excision is one of the most important factors for improvement of disease-specific survival. 27. All of the following are true regarding phyllodes tumors of the breast, except: A. Malignancy is determined by the stroma. B. Treatment of choice is surgical excision with wide margins. C. Breast conservation is contraindicated. D. Compared to epithelial breast cancers, phyllodes tumors do not frequently metastasize to lymph nodes. 28. Which of the following is false regarding giant cell tumors? A. The majority are not malignant. B. There is >90% local control with surgery alone. C. Local control rates with radiation alone are poor. D. These occur most commonly at the epiphysis. 29. Osteosarcoma most commonly occurs in which part of the bone? A. Metaphysis. B. Diaphysis. C. Epiphysis. D. There is equal distribution throughout the bone. 30. Which is the second most common sarcoma in children? A. Osteosarcoma B. Ewing’s sarcoma

546

C. Binder et al.

C. Rhabdomyosarcoma D. Leiomyosarcoma 31. Which is the most important prognostic factor in clear cell sarcoma? A. Margin status B. Lymphovascular invasion C. Grade D. Size 32. Any node-positive sarcoma is AJCC 8th edition stage: A. IIA B. IIB C. III D. IV 33. A 7 cm soft tissue sarcoma of the thigh with invasion of the superficial fascia is not found on CT scan to involve the deep musculature. By AJCC 8th edition staging, this would be considered: A. T1 B. T2 C. T3 D. T4 34. The 2005 Princess Margaret study by White et  al. for high-grade soft tissue sarcomas of the extremity found malignant cells up to how far beyond the tumor margin? A. 1 cm B. 2 cm C. 4 cm D. 5 cm 35. All of the following are true for pre-op vs post-op radiation for soft tissue sarcomas of the extremity except: A. Pre-op RT has higher rates of wound healing complications for lower extremity tumors. B. Post-op RT has worse long-term complications. C. In the absence of positive margins, pre-op RT total dose is less than post-op RT total dose. D. Post-op RT has superior overall survival. 36. Which of the follow is least likely to spread to a lymph node? A. Clear cell sarcoma B. Epithelioid sarcoma C. Rhabdomyosarcoma D. Alveolar sarcoma

13  Soft Tissue Sarcomas and Bone Tumors

547

37. According to the Red Journal Consensus Guidelines for preoperative radiation treatment of retroperitoneal sarcomas, all of the following are true except: A. CTV is the GTV + 1.5 cm. B. The CTV should be cropped away from the bone, kidney, liver, and bowel. C. 4D motion assessment is not necessary. D. PTV is the CTV + 5-12 mm. 38. Which of the following are acceptable GTV and CTV volumes for neoadjuvant treatment of a 9 cm high-grade soft tissue sarcoma of the extremity? A. GTV – T1 enhancing tumor + T2 enhancing tissue, CTV – GTV + 3 cm longitudinally and 1.5 cm radially B. GTV – T1 enhancing tumor, CTV – GTV + 3 cm longitudinally and 1.5 cm radially + T2 enhancing tissue C. GTV – T1 enhancing tumor + T2 enhancing tissue, CTV – GTV + 2 cm longitudinally and 1 cm radially D. GTV – T1 enhancing tumor, CTV – GTV + 2 cm longitudinally and 1 cm radially + T2 enhancing tissue 39. Which of the following is not true of the NCI Rosenberg et al. study published in 1982 that prospectively compared amputation to limb-sparing surgery with radiation for soft tissue sarcomas of the extremity? A. In the limb-sparing surgery group, at-risk areas received 50Gy, and the tumor bed received 60–70 Gy. B. There was no difference in overall survival. C. Local control was better with amputation. D. All patients received adjuvant chemotherapy. 40. The Musculoskeletal Staging Tumor Society (MSTS) staging system (also known as Enneking’s surgical staging system) is often used for osteosarcoma staging. Which of the following is not correct? A. Stage I – low grade. B. Stage IV – metastatic. C. Stage II – high grade. D. Nodal involvement is not included in MSTS staging. 41. For osteosarcoma patients who are eligible for surgery but refuse, response to radiation treatment is predicted by response to induction chemotherapy. What 5-year overall survival is expected after radiation for patients who respond to induction vs those who do not? A. 90% vs 35% B. 61% vs 20% C. 82% vs 70% D. 50% vs 10%

548

C. Binder et al.

42. Which of the following was a finding from the retrospective review of extremity soft tissue sarcomas by Folkert et al. from JCO 2014 that compared neoadjuvant/adjuvant radiation using conventional techniques vs IMRT? A. Conventional radiation had improved 5-year local recurrence but worse acute skin toxicity. B. IMRT had improved 5-year local recurrence but worse chronic toxicity. C. IMRT had improved median time to local recurrence and less acute skin toxicity. D. Conventional radiation had a higher 5-year local recurrence and worse acute skin toxicity. 43. Which of the following is NOT true about RTOG 0630 (multi-institutional phase II trial for pre-op IGRT to a reduced volume)? A. All patients received chemotherapy. B. The primary end point was late radiation morbidity. C. Results were compared to the preoperative arm of the CAN-NCIC-SR2 study (Davis et al. Late radiation morbidity following randomization to preoperative vs postoperative radiotherapy in extremity soft tissue sarcoma. Radiother Oncol 2005). D. 3D or IMRT planning was allowed. 44. In RTOG 0630, what were the treatment volumes? Pick all that are true. A. The GTV included post-contrast T1 enhancement and T2 signal. B. The CTV included suspicious edema but the GTV did not. C. GTV to CTV expansion for intermediate- to high-grade tumors ≥8 cm was 1.5 cm radially and 3 cm longitudinally. D. GTV to CTV expansion for low-grade tumors or tumors 60 was the only factor that predicted local recurrence. Margin status (positive margins) did not predict for recurrence. Pisters PW, et  al. Long-term results of a prospective randomized trial of adjuvant brachytherapy in soft tissue sarcoma. J Clin Oncol. 1996;14(3):859–68. 17. The correct answer is C. NCI Canada trial • 190 of 266 planned patients • Randomization –– Pre-op external beam radiation therapy Dose: 50  Gy in 25 fractions → (if + surgical margins) 16–20 Gy boost –– Post-op radiation 50 Gy in 25 fractions +16–20 Gy boost • Results, pre-op vs post-RT, respectively: –– No difference in LC or RFS: 5-year LC: 93% vs 92% 5-year RFS 58% vs 59% –– Predictor for LR: positive surgical margins –– Predictors of RFS and OS: Size/grade of the primary tumor –– Toxicity Acute wound complications: 35% vs 17%

13  Soft Tissue Sarcomas and Bone Tumors

555

Late grade 2 fibrosis: 32% vs 48% Edema: 15% vs 23% Joint stiffness: 18 vs 23% O’Sullivan B, et al. Five-year results of a randomized phase III trial of preoperative versus post-operative radiotherapy in extremity soft tissue sarcoma. J Clin Oncol. 2004 ASCO Annual Meeting Proceedings. Vol 22, No 14S (July 15 Supplement). 2004;9007. Davis AM, et al. Late radiation morbidity following randomization to preoperative versus postoperative radiotherapy in extremity soft tissue sarcoma. Radiother Oncol. 2005;75(1):48–53. 18. The correct answer is D. American Brachytherapy Society Guidelines for soft tissue sarcoma • Ineligibility for brachytherapy. –– CTV cannot be adequately encompassed in implant geometry. –– Proximity of treatment volume near a critical structure precludes delivery of a meaningful dose. –– Resection margins are positive. • Skin ulceration does not preclude brachytherapy. Nag S, et  al. The American Brachytherapy Society recommendations for brachytherapy of soft tissue sarcomas. In J Radiat Oncol Biol Phys. 2001;49:1033–43. 19. The correct answer is C. • Preoperative radiotherapy dose: 50 Gy/25 fractions • Postoperative boost: 16–18 Gy for microscopic positive margins. 20–26 Gy for gross residual tumor • Postoperative radiotherapy: –– 50–60 Gy –– 66 Gy (including boost of 16 Gy) • Brachytherapy –– LDR: 45 Gy in 4–6 days with 0.45 Gy/h –– HDR: 36 Gy in BID fractions of 3.6 Gy in 5 days Pisters PW, et  al. Long-term results of a prospective randomized trial of adjuvant brachytherapy in soft tissue sarcoma. J Clin Oncol. 1996;14:859–68. O’Sullivan B, et  al. Preoperative versus postoperative radiotherapy in soft-­ tissue sarcoma of the limbs: a randomised trial. Lancet. 2002;359:2235–41. Nag S, et  al. The American Brachytherapy Society recommendations for brachytherapy of soft tissue sarcomas. In J Radiat Oncol Biol Phys. 2001;49:1033–43. Arthur DW, et al. Partial breast brachytherapy after lumpectomy: Low dose-rate and high dose-rate experience. Int J Radiat Oncol Biol Phys. 2003;56:681–9. 20. The correct answer is B. Soft tissue sarcoma • Follows anatomical fascial planes, which has implications in postsurgical management.

556

C. Binder et al.

• Radiotherapy volumes follow this behavior, with longer longitudinal margins than axial margins. • Superficial tumors are resected with the deep fascia as the deep margin. –– Postoperative radiation does not need to extend into the deeper muscle compartment. • A radiological-pathological series of 15 patients from Mt. Sinai and Princess Margaret Hospital –– Results: tumor cells are found at a maximum of 4 cm from the primary tumor, suggesting that the longitudinal CTV margin can be 4 rather than 5 cm. White LM, et al. Histological assessment of peritumoral edema in soft tissue sarcoma. Int J Radiat Oncol Biol Phys. 2005;61(5)1439–45. O’Sullivan B, et al. Target description for radiotherapy of soft tissue sarcoma. In: Gregoire V, Scaliest P, Ang KK, editors. Clinical target volumes in conformal radiotherapy and intensity modulated radiotherapy. Heidelberg: Springer; 2003. p. 205–27. 21. The correct answer is A. Translocations in sarcoma • t(2;13) and t(1;13): alveolar rhabdomyosarcoma • t(X;18): synovial cell sarcomas • t(12;22): myxoid liposarcoma • t(X;17): alveolar sarcoma of soft parts Kawai A, et al. SYTSSX gene fusion as a determinant of morphology and prognosis in synovial sarcoma. N Engl J Med. 1998;338:153–60. Latanya M, et al. Impact of SYT– SSX fusion type of the clinical behavior of synovial sarcoma: a multi-institutional retrospective study of 243 patients. Cancer Res. 2002;62:135–40. 22. The correct answer is B. DeLaney et al. neoadjuvant chemoradiation in STS • 48 patients with localized, high-grade, large (>8 cm) soft tissue sarcomas • Treatment timeline –– MAID chemo: mesna, Adriamycin, ifosfamide, and dacarbazine –– RT 22 Gy in 11 fractions –– MAID chemo: mesna, Adriamycin, ifosfamide, and dacarbazine –– RT 22 Gy in 11 fractions –– Surgery –– If + surgical margins: 16 Gy boost –– MAID × 3 cycles • Results –– Neoadjuvant chemoradiation Improved 5-year OS: 87% vs 58% historically Improved distant metastasis-free rate: 75% vs 44% historically (p = 0.0016)

13  Soft Tissue Sarcomas and Bone Tumors

557

DeLaney TF, et al. Neoadjuvant chemotherapy and radiotherapy for large extremity soft-tissue sarcomas. Int J Radiat Oncol Biol Phys. 2003;56:1117–27. 23. The correct answer is C. Isolated limb perfusion • Uses heart-lung machine to deliver chemo (melphalan and TNF-α) to the solely affected limb. • Multicenter trial in Europe –– Overall response rates: 76% in initially unresectable soft tissue sarcomas 71% of limbs preserved –– The patients chosen for isolated limb perfusion are those in whom conventional resection would have left a marked functional loss or amputation. • One series reports overall response rates of 95% for soft tissue sarcoma with a 61% complete response rate. Amputation was avoided in all cases. Grunhagen DJ, et al. Isolated limb perfusion with tumor necrosis factor and melphalan prevents amputation in patients with multiple sarcomas in arm or leg. Ann Surg Oncol. 2005;12:473–9. Eggermont AM, et  al. Current uses of isolated limb perfusion in the clinic and a model system for new strategies. Lancet Oncol. 2003;4:429–37. Grunhagen DJ, et al. TNF-based isolated limb perfusion in unresectable extremity desmoid tumors. Eur J Surg Oncol. 2005;31:912–6. 24. The correct answer is C. • Sarcoma Meta-analysis Collaboration –– Adjuvant doxorubicin-based chemotherapy Improved distant recurrence-free survival. Improved local recurrence-free interval. Improved overall recurrence-free survival. Did not improve overall survival: HR was 0.89 (0.76–1.03), and not significant (p  =  0.12), but represented a 4% absolute benefit at 10 years. • Subsequent meta-analysis was published by the Sarcoma Meta-analysis Collaboration in 2000. –– 14 trials of 1568 patients –– Doxorubicin-based adjuvant therapy Improved both local recurrence-free interval and distant relapse-free interval Improved overall recurrence-free survival → corresponding to a significant absolute benefit of 6–10% at 10 years Did not improve OS significantly though still improved by 4% HR 0.89 95% CI 0.76–1.03 Largest benefit for adjuvant chemotherapy STS extremities • 2004 meta-analysis

558

C. Binder et al.

–– 1953 patients in 18 trials was published. –– Doxorubicin-based adjuvant chemotherapy for localized, resectable soft tissue sarcoma Maintained local recurrence, distant recurrence, and overall recurrence benefit Odds ratio for overall survival • Doxorubicin: 0.84 (95% CI 0.68–1.03, p = 0.09) • Doxorubicin + ifosfamide: 0.56 (95% CI 0.36–0.85, p = 0.01) Sarcoma Meta-analysis Collaboration: adjuvant chemotherapy for localized resectable soft-tissue sarcoma of adults: meta-analysis of individual data. Lancet. 1997;350:1647–54. Sarcoma Meta-analysis collaboration: adjuvant chemotherapy for localized resectable soft-tissue sarcoma of adults. Cochrane Database Syst Rev. 2000;(4):CD001419. Pervaiz N, et al. A systematic metaanalysis of randomized controlled trials of adjuvant chemotherapy for localized resectable soft-tissue sarcoma. Cancer. 2008;113(3):573–81. 25. The correct answer is B. Retroperitoneal sarcomas • Comprise 10–15% of all sarcomas • Compared to extremity STS –– Presents later –– Larger in size due to the absence of symptoms • Most common histologies –– Liposarcoma –– Leiomyosarcoma • Primary therapy –– Surgical resection –– Less clear role for adjuvant or neoadjuvant therapy Adjuvant radiotherapy complicated by the dose-limiting adjacent bowel Neoadjuvant preoperative radiotherapy • Tumor displaces the dose-limiting bowel. Intraoperative radiotherapy studied at Massachusetts General • Electron boost after preoperative external beam • Improved LC: 83% vs 74% • Improved OS: 61% vs 30% Gieschen HL, et  al. Long-term results of intra-operative electron beam radiotherapy for primary and recurrent retroperitoneal soft tissue sarcoma. Int J Radiat Oncol Biol Phys. 2001;50:127–31. 26. The correct answer is A. Primary sarcomas of the stomach • Rare entity • Comprised of two major types –– Gastrointestinal stromal tumors (GIST) –– Leiomyosarcoma

13  Soft Tissue Sarcomas and Bone Tumors

559

• Management –– Complete excision is one of the most important factors for improved disease-­specific survival. • Recurrence patterns –– LR as the initial recurrence occurs in ~40% of both tumor types. –– Pulmonary metastases are much more common in leiomyosarcoma than GIST tumors. –– Conversely, GIST tumors present with hepatic metastases as the first site of recurrence in half of cases, compared to only 20% for leiomyosarcoma. • A phase II trial of imatinib mesylate × 2 doses in patients with metastatic or unresectable GIST showed partial response rates of 68%. Clary BM, et al. Gastrointestinal stromal tumors and leiomyosarcoma of the abdomen and retroperitoneum: a clinical comparison. Ann Surg Oncol. 2001;8:290–9. Demetri GD, et al. Efficacy and safety of imatinib mesylate in advanced gastrointestinal stromal tumors. N Engl J Med. 2002;347:472–80. 27. The correct answer is C. • Phyllodes tumors of the breast –– Tumors of both epithelial and stromal elements. –– Stromal environment determines malignancy. –– Rarely metastasize to regional lymph node in contrast to epithelial breast tumors. –– Treatment Standard therapy: surgical excision with wide margins. Breast-conserving surgery is a reasonable option. If close/+ margins → adjuvant radiation to reduce LR – most common recurrence site. Mangi AA, et  al. Surgical management of phyllodes tumors. Arch Surg. 1999;134:487–92; discussion 292–483. Rowell MD, et al. Phyllodes tumors. Am J Surg. 1993;165:76–379. Kapiris I, et al. Outcome and predictive factors of local recurrence and distant metastases following primary surgical treatment of high-grade malignant phyllodes tumours of the breast. Eur J Surg Oncol. 2001;27:723–30. 28. The correct answer is C. Giant cell tumors • Commonly (98–90%) nonmalignant. • These occur more commonly in those aged 20–30. • Occur in the epiphysis. • Treatment –– Primary initial management: Surgery alone Local control (>90%) –– For inoperable tumors or positive margins: Radiation alone (54 Gy)

560

C. Binder et al.

Alektiar KM.  Soft tissue sarcoma. In: Gunderson LL, Tepper JE, editors. Clinical radiation oncology. 4th ed. Philadelphia: Elsevier Saunders; 2016. p. 1355–91. 29. The correct answer is A. Osteosarcoma most frequently occurs in the metaphysis of the bone. Ewing’s sarcoma most frequently occurs in the diaphysis of the bone. Alektiar KM. Soft tissue sarcoma. In: Gunderson LL, Tepper JE, editors. Clinical radiation oncology. 4th ed. Philadelphia: Elsevier Saunders; 2016. p. 1355–91. 30. The correct answer is B. While rhabdomyosarcoma is the most common sarcoma in children, the second most common is Ewing’s sarcoma. Bishop MW, Krasin MJ. Pediatric soft tissue sarcomas. In: Gunderson LL, Tepper JE, editors. Clinical radiation oncology. 4th ed. Philadelphia: Elsevier Saunders; 2016. p. 1403–11. 31. The correct answer is D. Size is the most important prognostic factor in clear cell sarcoma. In Kawai A, et al. Clear cell sarcoma of tendons and aponeuroses: an analysis of 75 cases. J Clin Oncol. 2006 ASCO Meeting Proceedings Part I. Vol 24, No. 18S (June 20 Supplement):9572. 32. The correct answer is D. • Any N1M0 disease constitutes stage IV disease per AJCC 8th edition. • This is in contrast to the AJCC 7th edition which would have designated N1 disease as stage III. Yoon, et al. Soft tissue sarcoma of the trunk and extremities. AJCC Cancer Staging Manual (8th edition). Springer International Publishing: American Joint Commission on Cancer; 2017. 33. The correct answer is B. • AJCC 8th edition soft tissue sarcoma of the extremity T staging –– T0 – No evidence of primary tumor –– T1 – Tumor ≤5 cm in greatest dimension –– T2 – Tumor 5.1–10.0 cm in greatest dimension –– T3 – Tumor 10.1–15.0 cm in greatest dimension –– T4 – Tumor >15.0 cm in greatest dimension • Invasion of the superficial fascia no longer affects T staging. Yoon, et al. Soft tissue sarcoma of the trunk and extremities. AJCC Cancer Staging Manual (8th edition). Springer International Publishing: American Joint Commission on Cancer; 2017 34. The correct answer is C. White et al. histologic assessment of tumor cells beyond the primary tumor • 15 patients s/p resection without neoadjuvant therapy. • The presence of tumor cells in the surrounding tissue was determined histologically. –– 10/15 of patients had sarcoma cells beyond tumor margin. Within 1 cm (n = 6)

13  Soft Tissue Sarcomas and Bone Tumors

561

Up to 4 cm away (n = 4) 9/10 cases found in tissue with increased T2 MRI signal White LM, et al. Histologic Assessment of Peritumoral Edema in Soft Tissue Sarcoma. Int J Radiat Oncol Biol Phys. 2005;61(5):1439–45. 35. The correct answer is D. • NCI Canada pre-op vs post-op RT. Please refer to question 17 for further details. • MD Anderson –– Retrospective review of 517 extremity soft tissue sarcoma cases –– Results: Greater complications in post-op cases at 10 years • Osteoradionecrosis • Soft tissue necrosis • Bone fracture • Edema • Fibrosis –– No difference in 10-year local control, disease-specific survival, nodal relapse, or distance relapse was found. –– Fractionation was similar to the above Princess Margaret study. Zagars GK, et al. Preoperative vs. postoperative radiation therapy for soft tissue sarcoma: a retrospective comparative evaluation of disease outcome. Int J Radiat Oncol Biol Phys. 2003;56(2):482–8. O’Sullivan B, et al. Preoperative versus postoperative radiotherapy in soft-tissue sarcoma of the limbs: a randomised trial. Lancet. 2002;359(9325):2235–41. Davis AM, et al. Late radiation morbidity following randomization to preoperative versus postoperative radiotherapy in extremity soft tissue sarcoma. Radiother Oncol. 2005;75(1):48–53. 36. The correct answer is D. • Mnemonic SCARE historically identified sarcomas likely to spread to lymph nodes. • Altered to CARE after 2017 NCDB review examined nodal incidence soft tissue sarcomas. –– Patients treated from 2000 to 2014 Nodal incidence Synovial cell carcinoma: 4.1% Clear cell carcinoma: 23.5% Angiosarcoma: 10.5% Rhabdomyosarcoma: 21.4% Epithelioid: 19.4% Ashamalla M, et al. Clinical presentation and patterns of care in SCARE soft tissue sarcoma. Int J Radiat Oncol Biol Phys. 2017;99(2):E749. 37. The correct answer is C. Red Journal Consensus Guidelines for preoperative radiation treatment of retroperitoneal sarcomas

562

C. Binder et al.

• Target volumes if 4D motion is assessed (recommended for all upper abdominal tumors) –– iGTV: contour GTV incorporating 4D motion; this accounts for internal margin (IM). –– ITV = iGTV +1.5 cm (CTV expansion) for upper abdominal tumors. Edit ITV at interfaces: • Retroperitoneal compartment, bone, kidney, liver: 0 mm • Bowel and air cavity: 5 mm • Under skin surface: 3–5 mm according to institutional preference If tumor extends to inguinal canal, expand iGTV by 3 cm inferiorly. –– PTV With IGRT: ITV + 5 mm No IGRT: ITV + 9–12 mm • Target volumes if 4D motion is NOT assessed and tumor has a significant component below the pelvic brim –– GTV: contour gross tumor volume –– CTV = GTV + 1.5 cm for tumors below pelvic brim Edit CTV at interfaces: • Retroperitoneal compartment, bone, kidney, liver: 0 mm • Bowel and air cavity: 5 mm • Under skin surface: 3–5 mm according to institutional preference If tumor extends to inguinal canal, expand GTV by 3 cm inferiorly. –– PTV With IGRT: CTV + 5 mm No IGRT: CTV + 9–12 mm Baldini EH, et  al. Treatment guidelines for preoperative radiation therapy for retroperitoneal sarcoma: preliminary consensus of an international expert panel. Int J Radiat Oncol Biol Phys. 2015;92(3):602–12. 38. The correct answer is B. • Based on a 2010 RTOG consensus statement and RTOG 0630, the following are recommended margins for neoadjuvant RT for soft tissue sarcomas of the extremity: –– GTV: gross tumor defined by T1-weighted contrast-enhanced MRI –– CTV Intermediate- and high-grade STS ≥8 cm: GTV + 3 cm longitudinally +1.5  cm radially + suspicious edema defined by T2-weighted MRI Low-grade STS or STS 50,000/μL B. Eight-month-old with WBC count 95%. B. Prophylactic cranial irradiation is still indicated for high initial white count, T-cell phenotype, Ph + chromosome, and CNS-3 disease. C. Intrathecal chemotherapy has successfully replaced spinal radiation as part of CNS preventive therapy. D. Maintenance regimens include methotrexate and 6-mercaptopurine. 76. All of the following high-risk groups will be treated with prophylactic cranial irradiation, except: A. T-cell subtype B. Ph + chromosome C. 35 Gy Krasin MJ, et  al. Intensity-modulated radiation therapy for children with intraocular retinoblastoma: potential sparing of the boy orbit. Clin Oncol. 2004;16:215–22. Pradhan DG, et al. Radiation therapy for retinoblastoma: A retrospective review of 120 patients. Int J Radiat Oncol Biol Phys. 1997;39:

14  Pediatric Non-CNS Tumors

605

3–13. Kaste SC, et al. Orbital development in long-term survivors of retinoblastoma. J Clin Oncol. 1997;1183–89. Peylan-Ramu N, et al. Orbital growth retardation in retinoblastoma survivors: work in progress. Med Pediatr Oncol. 2001;37:465–70. Abramson DH, et al. Second non-ocular tumors in survivors of bilateral retinoblastoma. Ophthalmology. 1998;105:573–80. 48. The correct answer is C. Neuroblastoma • Derived from the adrenergic neural crest cells. • The most common extracranial solid tumor in children. • Median age of diagnosis is 17 months. • The most common site is the adrenal gland: 35%. Brodeur G, et al. Neuroblastoma. In: Pizzo P, Poplack D, editors. Principles and practice of pediatric oncology. 4th ed. Philadephia: Lippincott Williams and Wilkins; 2002. p.  895–937. Halperin E, Constine L, Tarbell N, Kun L.  Neuroblastoma. In: Pediatric radiation oncology. 3rd ed. Philadelphia: Lippincott Williams and Wilkins; 1999. p. 163–203. 49. The correct answer is C. Unfavorable predictive factors in neuroblastoma • MYCN –– Occurs in 25% of neuroblastomas overall 5–10% of low-stage neuroblastoma 30–40% of advanced disease • Near-diploid or pseudodiploid tumors –– Diploidy or pseudodiploidy frequently occurs with MYCN amplification. • Deletion of 1p Fong CT, et al. Loss of heterozygosity for the short arm of chromosome 1 in human neuroblastomas: correlation with N-myc amplification. Proc Natl Acad Sci USA. 1989;86:3753. Brodeur GM, et  al. Amplification of N-myc in untreated neuroblastomas correlates with advanced disease stage. Science. 1984;224:1121–4. Matthay KK.  Neuroblastoma: biology and therapy. Oncology. 1997;11:1857–66. 50. The correct answer is B. Neuroblastoma • Metastasizes to the lymph nodes, bone marrow, liver, and bone. • Predilection for bones of the skull. –– Posterior orbit → “raccoon eyes” from periorbital ecchymosis • Lung and brain metastases are rare. Wolden S, Roberts SS. Neuroblastoma. In: Gunderson LL, Tepper JE, editors. Clinical radiation oncology. 4th ed. Philadelphia: Elsevier Saunders; 2016. p. 1447–54. 51. The correct answer is D. Neuroblastoma • Children 1  year present more commonly (81%) with disseminated disease. • Opsomyoclonus is rare. –– Paraneoplastic symptom of myoclonic jerking and random eye movements is rare. • Excess catecholamine secretion. • Norepinephrine, vanillylmandelic acid, 3-methoxy-4-hydroxyphenylglycol, or homovanillic acid. • Rarely symptomatic. Wolden S, Roberts SS. Neuroblastoma. In: Gunderson LL, Tepper JE, editors. Clinical radiation oncology. 4th ed. Philadelphia: Elsevier Saunders; 2016. p. 1447–54. 52. The correct answer is D. Work-up for neuroblastoma • CT or MRI –– Image the primary site and lymph nodes –– Rule out distant metastatic diseases in other organs (liver) • MIGB and technetium –– Assess for bone metastases • Urine catecholamines • Bilateral iliac crest bone marrow biopsy Golding SJ, et al. The role of computed tomography in the management of children with advanced neuroblastoma. Br J Radiol. 1984;57:661. Fletcher BD, et al. Abdominal neuroblastoma: magnetic resonance imaging and tissue characterization. Radiology. 1985;155:699. Heisel MA, et  al. Radionuclide bone scan in neuroblastoma. Pediatrics. 1983;71:206. Voute PA, et al. Detection of neuroblastoma with [I-131] metaiodobenzylguanidine. Prog Clin Biol Res. 1985;175:389–98. Brodeur GM, et  al. Revisions in the international criteria neuroblastoma diagnosis, staging, and response to treatment. J Clin Oncol. 1993;11:1466. LaBrosse EH, et  al. Catecholamine metabolism in neuroblastoma. J Natl Cancer Inst. 1976;57:633–43. 53. The correct answer is C. • As per the International Neuroblastoma Staging System –– Stage 4: metastatic to the lymph nodes and other organs like the liver, bone, and >10% bone marrow –– Stage 4S: the original tumor is located only where it started with spread limited to the skin, liver, and/or bone marrow (50 chromosomes). J Clin Oncol. 1000;18:1876–87. Brodeur GM, et al. Near-haploid

14  Pediatric Non-CNS Tumors

613

acute lymphoblastic leukemia: a unique subgroup with a poor prognosis? Blood. 1981;58:14–19. Heerema NA, et  al. Hypodiploidy with less than 45 chromosomes confers adverse risk in childhood acute lymphoblastic leukemia: a report from the children’s cancer group. Blood. 1999;94:4036–45. 72. The correct answer is C. Translocations in ALL • t(12;21) chromosomal translocation –– Most commonly identified: 16–22% –– Associated with a favorable prognosis • t(9;22)(q34;q11) translocation –– Ph chromosome –– 5% of childhood ALL –– Has the worst prognosis • t(8;14) – unfavorable prognosis • t(4;11) – unfavorable prognosis Chromosomal abnormalities and their clinical significance in acute lymphoblastic leukemia. Third international workshop on chromosomes in leukemia. Cancer Res. 1983;43:868–73. Bloomfield CD, et al. Chromosomal abnormalities identify high-risk and low- risk patients with acute lymphoblastic leukemia. Blood. 1986;68:205–12. Nowell P. Molecular monitoring of pre-B acute lymphoblastic leukemia. J Clin Oncol. 1987;5:692. Ribeiro RC, et al. Clinical and biologic hallmarks of the Philadelphia chromosome in childhood acute lymphoblastic leukemia. Blood. 1987;70:948–53. 73. The correct answer is D. ALL presentation • Signs and symptoms relate to the extramedullary involvement and bone marrow infiltration. • Fever, lymphadenopathy, and hepatomegaly (two-thirds of patients) are common. • CNS symptoms are rare, as less than 5% of children present with CNS involvement. Bleyer WA.  Central nervous systemic leukemia. Pediatr Clin North Am. 1988;35:789–814. 74. The correct answer is B. ALL risk groups. Of the listed subgroups, the 8-month-old has the worst prognosis. • Low –– Age 2–9. –– WBC